| 注册
home doc ppt pdf
请输入搜索内容

热门搜索

年终总结个人简历事迹材料租赁合同演讲稿项目管理职场社交

1378国开电大本科《管理英语3》形考任务(单元自测1至8)试题及答案(精华版)

h***s

贡献于2022-02-11

字数:88763

1378国开电科理英语3形考务(单元测18)试题答案
(精华版)
说明:题目机快捷键查找功搜索试题答案资料国开台形考期末纸质考试必备资料
单元测1 形考务试题答案
选择填空
[题目]―___________________
―NosorryI didn't know the rule about punching in
[答案]Didn't you punch in this morningWendy?
附:
[题目]―___________________
―起知道卡规
[回答]温迪天早没卡?
[题目]―I suppose there'll be a lot of arguments
―__________________
[答案]I should imagine so
附:
[题目]―想会争
―__________________
[答案]应该样想
[题目]―I wish you success in your career
―_________________
[答案]The same to you
附:
[题目]―祝事业成功
―_________________
[回答]样
[题目]―I'm leaving for Shanghai tomorrow
―__________________
[答案]Have a pleasant trip
附:
[题目]―明天海
―__________________
[回答]旅途愉快
[题目]―Who should be responsible for the accident?
―The bossnot the workersThey just carried out the order_______
[答案]as told
附:
[题目]―谁应该事负责?
―老板工执行命令_______
[答案]说
[题目]―___________________
―We have three major divisions:OperationsSales and MarketingFinance and Administration
[答案]How many departments do we have?
附:
[题目]―___________________
―三部门:运营销售营销财务行政
[回答]部门?
[题目]A bus driver_________the safety of his passengers
[答案]is responsible for
[题目]A number of boys_________absent some time during the term
[答案]have been
[题目]He will write to me as soon as he_________home
[答案] returns 
[题目]I am_________your early coming
[答案]looking forward to
[题目]Leave the reference books behind_________you won't be able to think independently
[答案]or
[题目]My friend invited me_________the art cluband I accepted it with pleasure
[答案]to join
[题目]The key_________successful implementation is clearly communicating the strategy to the whole company
[答案]to
[题目]The programme gives students the_________to learn more about global warming
[答案]opportunity
[题目]This is the man_________last night
[答案]whom I saw
二翻译
面句子选择正确翻译(题10分)
操作提示:通题干拉框选择题目正确答案
1If you are smart enoughand have got ambition and keep pushing forwardyou can rise to the top of your chosen profession
[答案]C果聪明进心勇直前选职业领域中出类拔萃
2It's up to you to strive for your placeto keep yourself engaged and productive during a working life that may last around 50 years
[答案]B约50年职业生涯中否积极进勤奋工作取成效取决
3They are so unusual both in their talents and their accomplishments as to be considered rare exceptions
[答案]A正智成非样例子少少
4We will have to place ourselves where we can make the greatest contribution
[答案]C应该放做出贡献位置
5Some are still nervous about losing their jobs despite the low wages
[答案]A工资高担心失份工作
二听力理解
听话录音根话容判断正误(题10分)
task4 testmp3
操作提示:句子正确选择拉选项框T句子错误选择拉选项框F
1Wendy punched in this morning[答案]F
2They need to punch out after work[答案]T
3They need to work overtime[答案]T
4They can wear casual clothes when going out on a sales call[答案]F
5They can use office phones for personal matters[答案]F
附:
1温迪天早[回答]F
2班需卡[回答]T
3需加班[回答]T
4出做推销员时穿休闲装[答案]F
5办公电话处理事务[答案]F
二听力理解 听话录音根话容完成方题目(题10分)
TASK3 TESTMP3
操作提示:通题干拉框选择题目正确答案
1The two speakers are_________
[答案]Bcolleagues
2There are divisions in the company_________
[答案]A3
3Customer Service is a part of division_________
[答案]BSales and Marketing
4The Sales and Marketing includes departments_________
[答案]C4
5The director of the Sales and Marketing is now_________
[答案]Con a business trip
附:
1两位发言者_________
[答案]B事
2公司部门_________
[答案]A3
3客户服务部门_________部分
[答案]B销售营销
4销售市场包括部门_________
[答案]C4
5销售市场总监现_________
[答案]C出差
二完型填空(空10分)
操作提示:通题目中拉选项框选择恰词语补全填空
  Joachim's vision is based on an idea called pleaching (编织)where tree branches are grown so that they naturally weave togetherSince the growth patterns of trees are{1}by wind and sunlightit may be possible to control the way a tree develops
  These Fabricated Tree House Habitats would use trees grown into shapes as housingOne of the{2}of these designs is that trees would not have to be cut down for lumber
  A 100 percent tree house would take years to createJoachim said.{3}the climatea house could take anywhere from 5 to 30 years to growFortunatelythere's a way to speed up the processJoachim suggests including ecological materials such as sod (草皮)grasses and living branches in the housing designsThis material would be able to move as the house growsJoachim said.
  A home would become an actual ecosystema community of plantsanimalsand bacteria working togetherThe trees would also give off water vapor that would assist in{4}the homesSolar panels and wind would help provide energyThe tree homes might even have soil pocketswhere plants could grow from the structure itself
  Work has already begun on Joachim's first designa house made from 50 percent recycled and 50 percent living thingsJoachim is confident about the{5}of his workas he uses natural products without destroying nature
The environment and its study are very importantWe need to respect naturedon't take it for grantedhe said.
附:
约阿希姆愿景基种做pleching(编织)想法种植树枝便然编织起树木生长模式{1}受风阳光影响控制树木生长方式发展
预制树屋栖息长成形状树木作房屋中种设计{2}必木材砍伐树木
约阿希姆说建造座100 树屋需数年时间{3}气候座房子需530年生长幸运种方法加快进程约阿希姆建议包括生态材料例约阿希姆说房屋设计中草皮(草皮)草活树枝种材料着房屋生长移动
家成真正生态系统植物动物细菌工作社区树木会释放出水蒸气房屋提供帮助太阳电池板风助提供量树屋甚土壤袋植物结构身生长
约阿希姆第设计工作已开始座50回收材料50生物制成房子约阿希姆工作充满信心{5}天然产品破坏然
环境研究非常重尊重然想然说
1[答案]affected
2[答案]advantages
3[答案]Depending on
4[答案]cooling
5[答案]importance
二阅读理解
阅读面文章根文章容完成相应选择题(题10分)
ORGANIZATION STRUCTURE IN HOOGLE ENGINEERING
  I'm Michael BushManaging Director of Hoogle EngineeringI am pleased to welcome you here to our website and I'd like to tell you a little about the company and its organizationHoogle Engineering was set up in 1960It was divided into several departments at that timesuch as the sales departmentmarketing departmentand production departmentLots of managers were employed to manage it all
  Fortunately things are different nowSixty people are employed by Hoogle and communication between departments is considered to be one of the most important aspects of the businessThe market is global so we need to make contact with customers worldwidenot just locally
  But in the old days we were all in different departments and never spoke to each otherWe had a tall structureTraditionally we had people at the first level on the shop floormanufacturing products according to the instructions which they were givenThen you had a supervisory level of people who supervised them every dayThen you moved up to the middle managementwho were doing the tasks of getting new businessand then you had the senior management teamand then you had the boardwho decided the business strategySo there were a lot of levels in the company in the old days actually
The structure today is that we form teams within teams to place people who can manufacture a productEach team has members that can manufacture different productsThe actual teams now are selfmanagingso we don't even have team leadersYou've got the teamsand then you've got two peopleonly two peoplewho are what you think of as managementThis is generally called flat structure
附:
霍格尔工程中组织结构
霍格尔工程董事总理迈克尔•布什高兴欢迎您网站想告诉您关公司组织信息霍格尔工程成立1960年分割时分成部门销售部市场部生产部雇佣理理切
幸运现情况霍格尔雇60名员工部门间沟通认业务中重方面市场全球性需全球客户联系仅仅客户
部门互相交谈高结构传统车间第层指示制造产品然监督级天监督然升中层理员负责获新业务务然高级理团队然董事会决定营战略公司实际层次
天结构团队中组建团队安排制造产品员团队制造产品成员现实际团队理甚没团队领导团队然两两认理员通常称扁结构
操作提示:通题干拉框选择题目正确答案
1Hoogle Engineering was set up in_________
[答案]Bthe twentieth century
2There are many different_________of management in a tall structure
[答案]Blevels
3People in a flat structure usually work in_________
[答案]Cteams
4All of the following statements are true according to the passage EXCEPT that_________
[答案]Bthe author thinks the two structures have the same functions
5The best title for the passage is_________
[答案]AOrganization Structure in Hoogle Engineering
附:
1霍格尔工程成立_________
[答案]B二十世纪
2高层建筑种理方式
[答案]B水
3扁结构通常_________工作
[答案]C团队
4_________外根段落陈述正确
[答案]B作者认两种结构具相功
5篇文章佳标题_________
[答案]A霍格尔工程中组织结构
二阅读理解
阅读面文章根文章容判断文章句子正确(T)错误(F)(题10分)
MANAGING ONESELF
  We live in an age full of opportunities:If you are smart enoughand have got ambition and keep pushing forwardyou can rise to the top of your chosen professionno matter where you started out
  But with opportunity comes responsibilityCompanies today aren't managing their employees' careersProfessional workers must be their own chief executive officers (CEO)It's up to you to strive for your placeto keep yourself engaged and productive during a working life that may last around 50 yearsTo do those things wellyou'll need to have a deep understanding of yourselfnot only what your strengths and weaknesses arebut also how you learnhow you work with otherswhat your values areand where you can make the greatest contributionbecause only when you operate from strength can you achieve true excellence
History's great achieversNapoléondaVinciand Mozarthave always managed themselvesBut they are so unusual both in their talents and in their accomplishments as to be considered rare exceptionsNowmost of useven those of us with modest talentswill have to learn to manage ourselvesWe will have to learn to develop ourselvesWe will have to place ourselves where we can make the greatest contributionAnd we will have to stay mentally alert and engaged during a 50year working lifewhich means knowing how and when to change the work we do
附:

生活充满机遇时代:果足够聪明抱负断进取里开始登选择职业顶端
机遇会持续50年左右做事情需深入解―仅解优势劣势解学方式合作方式价值观什里做出贡献实力营做真正卓越
历史伟成者―破仑达芬奇莫扎特―总理华成寻常认罕见例外现中数天赋高必须学会理必须学会发展必须放做出贡献方50年工作生涯中必须保持头脑清醒投入意味着知道时改变做工作
操作提示:句子正确选择拉选项框T句子错误选择拉选项框F
1Companies today are responsible for employee's career[答案]F
2It is the CEO who decides your place[答案]F
3You need to understand your company well so that you can do things well[答案]F
4Understanding yourself means knowing well about your strengthsweaknessesyour valueshow you learnhow you do with others and so on[答案]T
5Ordinary people cannot manage themselves well[答案]F
附:
1天公司员工职业生涯负责[答案]F
2CEO决定位置[答案]F
3解公司事情做[回答]F
4解意味着解长处短处价值观学方式相处方式等等[答案]T
5普通理[回答]F
单元测2 形考务试题答案
选择填空
[题目]―_________father took part in the charity activity in the neighbourhood yesterday?
―Peter's
[答案]Whose
附:
[题目]―_________父亲昨天参加附慈善活动?
—彼
[答案]谁
[题目]I heard that you really had a wonderful time at John's birthday party_____?
[答案]didn't you 
[题目]_________important it is for kids to imagine freely
[答案]How
[题目]―SorryI made a mistake again
―_________Practise more and you’ll succeed
[答案]Never mind 
附:
[题目]—起犯错误
—_________练会成功
[答案]没关系
[题目]―__________________
―She gave two weeks
[答案]How much time did she give?
附:
[题目]—__________________
—两星期
[回答]少时间?
[题目]―Could I help you with your heavy box? You are so tired   
―_________I can manage it
[答案]Nothanks 
附:
[题目]―帮您搬运沉重箱子?您累
—_________做
[答案]谢谢
[题目]―How did you find your visit to QingdaoJoanna?
―__________________
[答案]Oh wonderful indeed
附:
[题目]—乔安娜发现次青岛行?
—__________________
[答案]哦确实棒
[题目]―I'm more than happy to go out this weekend
―_________We've got so much work to do at that time
[答案]Forget it 
附:
[题目]—周末高兴出玩
—_________时工作做
[答案]算
[题目]He gave up his study in college in_________
[答案]despair
[题目]I took it for_________that you wouldn't come here again
[答案]granted
[题目]If you need further informationplease_________our office
[答案]contact
[题目]It's_________to point to or talk about strangers in public
[答案]not polite
[题目]The_________was only sentenced to pay a fine of 10000
[答案]criminal
[题目]The light is too_________for me to readI can not stand any more
[答案]dim
[题目]Under no circumstance_________to tell lies to parents
[答案]are children allowed
二翻译
面句子选择正确翻译(题10分)
操作提示:通题干拉框选择题目正确答案
1Though they have a lot of applicantsthey can't find the people they want
[答案]B然应聘者找合适
2Practising more is the key to leaning a language
[答案]A学语言关键练
3Some are still nervous about losing their jobs despite the low wages
[答案]A工资高担心失份工作
4The teacher pointed out the grammar mistakes in the writing
[答案]C老师指出作文里语法错误
5How will the evaluation of these investments be carried out?
[答案]B投资进行评估呢?
二听力理解
听话录音根话容判断正误(题10分)
TASK4 TESTMP3
操作提示:通题干拉框选择题目正确答案
1What is the possible relationship between the two speakers?
[答案]BColleagues
2What is the opinion of the female speaker about high salaries?
[答案]CShe thinks high salaries stop people from moving on
3Who controls overtime work?
[答案]BThe workers
4FAIR in fair wages means_________
[答案]Ageneral
5The topic of the dialog is about_________
[答案]Chow to motivate the employees
附:
1两说话者间什关系?
[答案]B事
2女播高薪法?
[答案]C认高薪阻止继续前进
3谁控制加班?
[答案]B工
4公工资中公意味着_________
[答案]A般
5话题关_________
[答案]C激励员工
二完型填空(空10分)
操作提示:通题目中拉选项框选择恰词语补全填空
  When it comes to selecting candidates through interviewmore often than not the decision is made with the first five munutes of a meetingYet employers like to{1}themselves that they are being exceptionally thorough in their selection processesIn today's marketplacethe{2}of staff is fundamental to the company's success andas a resultrecruiters use all means at their disposal to secure the best in the field
  One method in particular that has risen in popularity is testingeither psychometric testing(心理测试)which attempts to define psychological characteristicsor aptitude testingThe idea is that those testing provide an organization with an extra way of establishing a candidate's suitability for a role
The employment of psychometric or ability testing as one{3}of the recruitment process may have some meritsbut in reality there is simply no real{4}The answer to this problem is experience in interview techniques and a strong definition of the elements of each position to be filledThe instinctive decision that many employers makebased on a CV and the first five minutes of a meetingare perhaps no less valid than any other tool employed in the{5}of recruitment
附:
通面试选择候选时通常会会议前五分钟做出决定雇喜欢{1}选择程中非常彻底市场中{2}员工数量公司成功基础招聘员会确保该领域佳
种特流行方法测试者心理测试(心理测试)试图定义心理特征者力测试想法测试组织提供种额外方法确定候选否适合角色
心理测量力测试作招聘程{3}优点实际根没真正{4}问题答案面试技巧方面验素明确定义许雇根简历会议前五分钟做出决定招聘中工具样效{5}
1[答案]convince
2[答案]quality
3[答案]component
4[答案]measure
5[答案]business
二阅读理解
阅读面文章根文章容完成相应选择题(题10分)
HUMAN RESOURCES MANAGEMENT
  Human Resources Management(HRM)is the process of managing people and their relationships in an organizationThese two processes are important in the success and growth of a business
  In managing peoplestaffing is the most important component of HRMIt needs to make guidelines and procedures for hiring and placementStaffing also involves the management of employees on matters like monitoring of holidaysabsenceshealth and safetydisciplinary actionand dismissal
  Another important element of HRM in managing people is keeping the employees in the companyThe training of employees to enable them to improve their career development is very importantGood HRM policies ensure that there are clear regulations which show the pay levels for the different positions in an organizationIt is important for the staff to know the career pathpaytraining and other opportunities that are available to themPerformance management is another important component of HRMThe reason for this is that many employers use it to evaluate career improvement and to determine pay increases
  The secondary role of HRM is the management of the relationships among people in an organizationThis includes staff within departments as well as across the whole organizationThe relationship between staff and management is an important factor in the success of an organizationIt decides how fast an organization will realize their goals
The focus of HRM is the people within an organizationRegular planningmonitoring and evaluation are important for the success of HRMSuccessful management ensures that all employees know their rolecareer development and also feel part of an organization
附:
力资源理
力资源理(HRM)组织中理员关系程两程企业成功发展非常重
员理中员配备力资源理重组成部分需制定招聘安置指导方针程序员配备涉员工假期缺勤健康安全纪律处分解雇等事项理
力资源理事理中重元素员工留公司员工进行培训促进职业发展非常重良力资源理政策确保明确规定显示职位薪酬水组织中重员工解职业道路薪酬培训机会绩效理力资源理重组成部分原许雇评估职业生涯改善确定加薪
力资源理次作理组织中员间关系包括部门整组织员工员工理层间关系组织成功重素决定组织快实现目标
力资源理重点组织定期规划监控评估力资源理成功非常重成功理确保员工解角色职业发展感受组织部分
操作提示:通题干拉框选择题目正确答案
1_________isare important in the success of a business
[答案]BThe process of managing people and their relationships
2The most important component of HRM is_________
[答案]Astaffing
3According to the passageit is important for the employees to know the career pathpaytraining and other opportunities through_________
[答案]Bpolicies
4We can use_________to evaluate career improvement and to determine pay increases
[答案]Cperformance management
5_________is NOT mentioned in the text about the success of HRM
[答案]CTraining
附:
1_________企业成功重
[答案]B理际关系程
2力资源理重组成部分_________
[答案]A员配备
3根文章员工通_________解职业道路薪酬培训等机会重
[答案]B政策
4_________评估职业发展确定加薪
[答案]C绩效理
5_________文中没提力资源理成功
[答案]C培训
二阅读理解
阅读面文章根文章容判断文章句子正确(T)错误(F)(题10分)
WAR FOR TALENT GOES ON IN JOB MARKET
  The war for talent reads like headlines from many years agobut it has never gone awaysays Eleanor Nickersondirector of UK operations for Top EmployersMany companies they have researched are short of talentsThough the companies may receive many applicationsthey cannot find the people they want
  Top Employers' research shows that offering good career chances is the key to attracting and keeping the talentSmart employees know their own value and will want to know what their employers can offer them after 5 or 10 years' career developmentSokeeping staff is the biggest challenge employers face in the long run
  Yet not every employee feels that they can better their career chancesSome are still nervous about losing their jobsdespite a recent fall in unemploymentsays the Trades Union Congress (TUC)TUC points out that some fourfifths of new jobs created have been in part of the economy where average pay rates are less than ?8 an hour since the recession beganMany of these jobs are on temporary or zerohours contracts
  A report from the Office for National Statistics published in February showed that real wages have been falling consistently since 2010It's the longest period since at least 1964We're still in the hardest living standards squeeze for over a century and those who are already working have had years of realterms pay cutssays TUC spokesperson Liz ChinchenUnderstanding the pressures that staff face is a good starting point for any employerIf employers want to show concern for their staffthey should be paying them well and understand that zerohours contracts bring insecurity and extreme money worries
附:
争夺战业市场继续
杰出雇英国业务埃莉诺•尼克森(Eleanor Nickerson)说争夺战读起年前头条新闻未消失研究许公司缺乏公司会收申请找想
杰出雇研究表明提供良职业机会吸引留住关键聪明员工知道价值会想知道5年10年职业发展雇提供什留住员工雇长期面挑战
然员工觉改善职业机会工会会(TUC)表示失业率降然担心失工作TUC指出约五分四新工作岗位创造济衰退开始直处均工资率低时8欧元济体中中许工作时合零时合
国家统计局2月份发布份报告显示2010年实际工资直持续降少1964年长段时期处世纪艰难生活水挤压中已处TUC 发言丽兹钦臣表示工作历年实际减薪解员工面压力雇说起点果雇想表达员工关心应该薪水理解零时合会带安全感极度金钱担忧 
操作提示:句子正确选择拉选项框T句子错误选择拉选项框F
1According to research by Top Employersmany corporations have enough talents for their development[答案]F
2The data shows the key to attracting talents is to offer high salary[答案]F
3The biggest challenge for employers to is keeping staff in the long run[答案]T
4Not all employees believe that they can get a better offer[答案]T
5The wages have been rising consistently since 2010[答案]F
附:
1根杰出雇研究公司足够发展[答案]F
2数显示吸引关键提供高薪[答案]F
3雇面挑战长期留住员工[答案]T
4员工相信更offer[答案]T
52010年工资直涨[答案]F
单元测3 形考务试题答案
选择填空
[题目]―_________Everything is going smoothly
[答案]What do you think of everything?
附:
[题目]—_________切进展利
[回答]觉切?
[题目]―Are you going on holiday for a long time?
―__________________
[答案]It was a long time
附:
[题目]—长期度假?
—__________________
[回答]久
[题目]―I wonder if I could use your computer tonight?
―_________
[答案]Surego ahead
附:
[题目]—想知道晚电脑?
—_________
[答案]然继续
[题目]―In what form will you take the investment?
―__________________
[答案]We'll contribute a site and the required premises
附:
[题目]—什形式进行投资?
—__________________
[答案]提供站点需场
[题目]―Is it more advisable to upgrade our present facilities than taking the risk of opening a new park?
―__________________
[答案]I don't think so
附:
[题目]—升级现设施否险开新公园更取?
—__________________
[答案]认
[题目]―Is it possible for you to expand business there?
―__________________
[答案]YesI think so
附:
[题目]—里拓展业务?
—__________________
[回答]想
[题目]A budget is an estimation of the_________and_________over a specified future period of time
[答案]revenue expenses
[题目]If the building project_________by the end of this month is delayedthe construction company will be fined
[答案]to be completed
[题目]John as well as Mike_________just been back from an important meeting
[答案]has
[题目]Let's discuss these problems at the meeting_________?
[答案]shall we
[题目]Purchasing the new production line will be a_________deal for the company
[答案]profitable
[题目]Rentelectricity and insurance are all examples of the company's_________
[答案]overhead
[题目]The manager urged his staff not to_________the splendid opportunity
[答案]miss
[题目]They are trying to_________the waste discharged by the factory for profit
[答案]exploit
[题目]They recommended that the amusement park_________
[答案]be extended
[题目]You can go out_________you promise to be back before 12 o'clock
[答案]as long as
二翻译
面句子选择正确翻译(题10分)
操作提示:通题干拉框选择题目正确答案
1The cost of the new project will go over budget
[答案]A新项目支出超预算
2How will the evaluation of these investments be carried out?
[答案]B投资进行评估呢?
3A credit card is an interest free shortterm loan
[答案]A信卡短期息贷款
4Assessing the data is a vital part of creating an effective marketing strategy
[答案]C数评估制定效市场营销策略重环
5We should discourage people from using their cars in the city centers
[答案]C应该劝阻市中心开车
二完型填空(空10分)
操作提示:通题目中拉选项框选择恰词语补全填空
  Many people who work in London prefer to live outside itand to go to their officesfactories or schools every day by train or by buseven though this means they have to get up earlier in the morning and{1}home later in the evening
  One benefit of living outside London is that houses are cheaperEven a small flat in London {2}a garden costs quite a lot to rentWith the same moneyone can get a little house in the{3}with a garden of his own
Thenin the country one can be free{4}the noise and hurry of the townAlthough one has to get up earlier and spend more time in trains or busedone can sleep better at nightandduring weekends and on summer eveningsone can enjoy the fresh clean air of the countryIf one likes gardeningone can spend one's spare time diggingplantingwatering and doing a lot of other jobs which are needed in the gardenThenwhen the flowers and vegetables come upone has the reward of a person who has shared the secrets of{5}
附:
许伦敦工作更喜欢住伦敦外天坐火车公汽车办公室工厂学校意味着必须早起早{1}晚点回家晚
住伦敦外处房子更便宜伦敦套公寓{2}花园租金相高样钱{3}花园房子
然乡{4}摆脱城镇喧嚣匆忙然必须早起花更时间火车公汽车晚周末夏天睡更 晚享受乡村清新空气果喜欢园艺利业余时间挖种浇水做花园里需工作然花蔬菜吧分享{5}秘密奖励
1[答案]arrive
2[答案]without
3[答案]country
4[答案]from
5[答案]nature
二阅读理解
阅读面文章根文章容完成相应选择题(题10分)
THREE STEPS TO A GOOD MARKETING BUDGET
  Without good budget managementyou are likely to overspend on marketing accidentallyHere are three steps that help to make a good marketing budget
STEP 1:ORGANIZE FINANCIAL INFORMATION
  The first step is to be clear of your current financial situationYou need to know how much money your company makes monthly and the changes that might arise in the futureAlthough income flows change throughout the yearyou must organize the information based on reliable revenuethe minimum amount of money your company makes each month
  A realistic budget plan will always focus on the income that exceeds the expensesnot the total revenue that comes inWhen you have determined the amount of disposable income available for the companyyou should determine where the money will go
STEP 2:DETERMINE WHERE YOU WANT TO SPEND MARKETING FUNDS
  After you know the total amount available to spend on marketingthe next step is how you plan to spend that moneyThree main factors decide how you spend marketing funds:the budget amountyour past experiencesand where you can reach the right audience
  You will start organizing how to spend the funds based on the amountBesidesyou need to consider the strategies that have worked in the pastAlsoyou need to consider which marketing channels will allow you to reach the right audience
STEP 3:ASSESS DATA AND MAKE APPROPRIATE CHANGES
  The final step in building a good marketing budget is the analysis of the plan and any changes that may improve revenueUltimatelymarketing is designed to bring in extra revenueIf the activity does not bring in any additional revenue over its costthen it is better to change strategy and try something else
Assessing the data is a vital part of creating an effective marketing strategyEvaluation begins with comparing past performance to the performance after marketing the product or servicesHowevera good budget alone is not enoughYou have to take action and remain committed to following through your planBy keeping the budget in mind when you make decisionsyou will have the opportunity to work out different strategies to find the best solutions for your business goals and prevent overspending on marketing
附:
制定良营销预算三步骤
果没良预算理您会意外营销超支三步骤助制定良营销预算
第步:整理财务信息
第步清楚目前财务状况需知道公司月赚少钱未出现变化然收入流年四季变化必须根信息组织信息收入—贵公司月低收入
现实预算计划始终关注超支出收入总收入您确定公司支配收入金额时您应该确定资金
第二步:确定您想营销资金处
解营销总金额步计划笔钱三素决定您营销资金:预算金额您验您里接触合适受众
您开始根金额组织资金外您需考虑行效策略外您需考虑营销渠道您接触合适受众
第三步:评估数进行适更改
建立良营销预算步分析计划提高收入变化终营销旨带额外收入果活动没带超成额外收入改变策略尝试方法
评估数制定效营销策略重组成部分评估始业绩营销产品服务业绩进行较然仅良预算够您必须采取行动继续致力遵循通您计划通做出决策时牢记预算您机会制定策略您业务目标找佳解决方案防止营销超支
操作提示:通题干拉框选择题目正确答案
1What should the budget plan be based on?
[答案]BThe company's reliable income
2What does disposable income mean in Passage 3?
[答案]CNet income
3You have to be clear of_________when organizing financial information
[答案]Athe income and expenses
4The three main factors that decide how you spend marketing funds are_________
[答案]Bthe budget amountyour past experiences and where you can reach the right audience
5Keeping the budget in mind when making marketing decisions will avoid_________
[答案]Cusing more than you earn
附:
1预算计划应该什?
[答案]B公司收入
2第3段中支配收入什意思?
[答案]C净收入
3整理财务信息时必须清楚_________
[答案]A收入支出
4决定您营销资金三素_________
[答案]B预算金额您验您里接触合适受众
5制定营销决策时牢记预算避免_________
[答案]C赚
二阅读理解
阅读面文章根文章容判断文章句子正确(T)错误(F)(题10分)
SAVE MONEY BY UNDERSTANDING WHY YOU SPEND IT
  To save moneyyou simply need to spend less than you earnIt is easy to saybut why is it so difficult to do? To answer this questionyou need to study the factors that keep you spending all the time
EASY ACCESS TO CREDIT
  For most peoplethis is the biggest reason for overspendingPeople can easily get a credit cardsimply by filling out a short form in a bank or jumping on their websiteThey can borrow money from banks to go shoppingeven buy cars or housesWhen you aren't physically handing someone money or a check for a purchaseyou will feel you are spending free money or you aren't spending money at allTry using cash to get your spending under control
MISUSING CREDIT CARDS
  A credit card is an interest free shortterm loanWhen used as plannedthis can be a powerful financial toolThe problem is when you begin to let the balance continue from month to monthIf you make a 200 purchase on your credit card and find out that at the end of the month you can't afford to repay the full amountthis is when the high interest rates on the cards really begin to hurt youThat is no way of saving money
GIVING IN TO TEMPTATION
  We all need to enjoy lifebut don't waste your financial future for a few guilty pleasures today if they aren't in your budgetWhen you know that you can't afford an activitydon't give inInsteadinvite your friends over for a home dinner partygame nightor some other activities where you can still enjoy time togetherbut without breaking your budget
SPENDING TO FEEL GOOD
Everyone enjoys getting something newbuying yourself something feels goodThere is absolutely nothing wrong with thisas long as you don't go overboardYou'll feel good about your purchase whether you make it with cash or by credit cardbut you'll feel even better when you don't have to spend the next two years trying to pay it off with 20 interest
附:
通解您什花钱省钱
想省钱花赚少说起容易做起什难?回答问题需研究直花钱素
轻松获信贷
数说超支原容易获信卡需银行填写张简短表格访问网站银行钱购物甚买车房屋您没亲某递钱购买支票时您会觉花钱根花钱尝试现金控制支出
滥信卡
信卡种免息短期贷款计划时成种强金融工具问题开始余额逐月持续时果信卡购买200美元卡发现月底起全部金额时候卡高利率真正开始伤害没办法省钱
屈服诱惑
需享受生活果您预算请罪恶乐趣浪费您财务未您知道负担起某项活动时请屈服相反邀请您朋友参加家庭晚宴游戏夜您然起享受时光会超出预算活动
花钱感觉良
喜欢新东西—买感觉东西绝没错走火入魔现金信卡购买会购买感觉良果您必接两年中试图 20 利息清您会感觉更
操作提示:句子正确选择拉选项框T句子错误选择拉选项框F
1You have to pay for the credit card loans by the end of each month[答案]T
2Using a credit card to pay is spending free money[答案]F
3It's easy to get a credit card but hard to make ends meet[答案]T
4If you suffer from a deficit this monthyou simply have to pay it off by the original amount in the future[答案]F
5Spending to feel good means keeping your spending within the budget[答案]T
附:
1您必须月底前支付信卡贷款[回答]T
2信卡支付花冤枉钱[答案]F
3办卡容易入敷出[回答]T
4果月亏需未原数额清[答案]F
5支出感觉良意味着您支出保持预算范围[答案]T
单元测4 形考务试题答案
选择填空
[题目]―How did you miss your train?
―__________________
[答案]WellI was caught in the traffic jam
附:
[题目]—错火车?
—__________________
[回答]吧堵车
[题目]―I'm dog tiredI can't walk any furtherTommy
―_________JennyYou can do it
[答案]Come on
附:
[题目]—累走汤米
—_________珍妮做
[答案]吧
[题目]―I've started my own software company
―__________________
[答案]No kidding Congratulations
附:
[题目]―创办软件公司
―__________________
[答案]开玩笑 恭喜
[题目]―It's getting darkI'm afraid I must be off now
―OK_________
[答案]See you
附:
[题目]―天快黑恐怕走
―行_________
[答案]见
[题目]―When do we have to pay the bill?
―_________December 30
[答案]By
附:
[题目]―什时候必须付账?
―_________12月30日
[答案]By
[题目]―Would you like some more beer?
―_________please
[答案]Just a little
附:
[题目]―想喝点啤酒?
―_________请
[答案]点点
[题目]Bring me a glass of beer_________?
[答案]will you
[题目]Can you tell me how_________to the railway stationplease?
[答案]to get
[题目]I_________to know how often the bus runs during rush hour
[答案]happen
[题目]I bought a new car last monthbut I_________my old car yet
[答案]have not sold
[题目]I didn't finish my homeworkNeither_________
[答案]did he
[题目]In Beijingthe best season in a year is probably_________autumn
[答案]late
[题目]Many factors both at home and abroad_________the bankruptcy of the multinational corporation
[答案]contributed to
[题目]Raising pigs_________my job at that time
[答案]was
[题目]We_________the last bus and didn't have any money for taxiso we had to walk home
[答案]missed
二翻译
面句子选择正确翻译(题10分)
操作提示:通题干拉框选择题目正确答案
1.We should discourage people from using their cars in the city centers
[答案]C应该劝阻市中心开车
2.Car owners may probably be against it at firstbut people who use public transportation will be in favor of it
[答案]B车开始会反公交通工具会支持
3.You must apply for approval to do roadwork and other roadrelated activities
[答案]B必须申请获批进行道路施工道路关活动
4.The first underground floor is set aside mainly for pedestrians and commercial facilitieswhile the second underground floor is planned for parking
[答案]A层行通道商业设施二层计划作停车场
5The area of grass should be kept short to deny pests a place to live
[答案]A长草区域应该害虫生存空间
二听力理解
听话录音根话容判断正误(题10分)
TASK3 TESTMP3
操作提示:通题干拉框选择题目正确答案
1How many people were fined for breaking traffic rules?
[答案]CThree
2Who got the biggest fine?
[答案]BMike's friend
3Helen was fined for_________
[答案]Aspeeding
4How long did Mike's friend have his driving license suspended?
[答案]BThree months
5Crackdown means_________
[答案]Crestricting undesirable actions
附:
1少违反交通规罚款?
[答案]C三
2谁罚单?
[答案]B迈克朋友
3海伦_________罚款
[答案]A超速
4迈克朋友驾吊销久?
[答案]B三月
5镇压意味着_________
[答案]C限制良行
二完型填空(空10分)
操作提示:通题目中拉选项框选择恰词语补全填空
It's very interesting to study names of different countriesChinese names are different from foreign namesOnce an English lady came to{1}meWhen I was introduced to her she saidGlad to meet youMiss PingThen she gave me her name card with three words on it:Betty JBlackSo I saidThank youMiss BettyWe looked at each other and laughed heartilyLater I found that the English people{2}their family names last and the given names firstwhile their middles are not used very muchI explained to her that the Chinese family name{3}firstthe given name lastso she{4}never call me Miss PingShe asked if we Chinese had a middle nameI told her we didn'tBut people may often find three words on a Chinese name cardIn this case the family name still comes firstthe other words after it{5}a twoword given nameIt is quite usual in ChinaMy sister is Li XiaofangShe has two words in her given name instead of just one like mine
附:
研究国家名字趣中文名字外国名字次位英国女士找{1}介绍时说高兴认识姐然 名片写着三字:贝蒂•布莱克说谢谢贝蒂姐互相眼开怀笑发现英国{2}姓氏姓氏 名字前中间解释说中国姓{3}前名字{4}姐问中国没中间告诉没中国常见妹妹李晓芳名字中两字样
1[答案]visit
2[答案]put
3[答案]came
4[答案]would
5[答案]is
二阅读理解
阅读面文章根文章容完成相应选择题(题10分)
BEIJING'S ROADS FOR THE FUTURE
  The road system in Beijing's central business district (CBD) is being redesigned to make public transportation easy and improve efficiency
  Plans for 45 roads within the CBD have been approvedmeasuring 415 kilometers in totalIn addition to public busesfree or low price shuttle buses from residence areas to subway stations will also be provided for commuters
  This is just over four times the number of roads opened in 2007and will add another 155 km to the existing road network
  Besides roadsan underground traffic system has also been operatingwhich includes four new subway lines beneath the CBD
  The underground connection system is made up of two levelsThe first underground floor is set aside mainly for pedestrians and commercial facilitieswhile the second underground floor is planned for parking
The city has also invested 140 million yuan to set up a traffic inquiry systemthrough which the public can check traffic conditions online and see the most convenient routes on their GPS systemIn the futurethe CBD will be fully covered by an intelligent traffic control system
附:
北京未路
北京中央商务区(CBD)道路系统正重新设计方便公交通提高效率
北京中央商务区45条道路规划已获批准总长415公里公巴士外通勤者提供住宅区铁站免费低价穿梭巴士
2007年开通道路数量四倍现道路网络中增加155公里
道路外交通系统已投入运营中包括北京中央商务区方四条新铁线
连接系统两层组成层行道商业设施预留二层规划停车场
该市投资14亿元建设交通查询系统市民通该系统线查询交通状况GPS系统查便捷路线未智交通全面覆盖北京中央商务区控制系统
操作提示:通题干拉框选择题目正确答案
1The road system in Beijing's downtown area is being to make public transportation easy and improve efficiency
[答案]Bdesigned again
2An underground system including new subway lines beneath the CBD has been operating
[答案]C4
3The underground connection system consists of_________level(s)
[答案]Btwo
4CBD stands for_________
[答案]ACentral Business District
5Beijing has set up_________for people to check traffic conditions
[答案]Ca traffic inquiry system
附:
1北京市中心城区道路系统正公交通变容易提高效率
[答案]B重新设计
2包括北京市中心城区新铁线系统已投入
[答案]C4
3连接系统_________层组成
[答案]B二
4北京市中心城区代表_________
[答案]A中央商务区
5北京设置_________供检查交通状况
[答案]C交通查询系统
二阅读理解
阅读面文章根文章容完成相应选择题(题10分)
ROAD CLOSURE PROCEDURES IN SYDNEY
  To minimize traffic jams in the city center and on major roads out of Sydneythe City's highway department will not agree to road closures scheduled for weekdays or SaturdaysOn minor roads outside the city centerweekdays and Saturdays may be consideredYou must apply for approval to do roadwork and other roadrelated activitiestree cutting and use of very large construction machineryladdersetcPlease note that approval is needed if you plan to block the road or footpath
  Applications to have a road closed to traffic must be completed and sent to the Department's Traffic Calming Committee at least a month before the scheduled date
  Roads can be closed partially or fullydepending on your requirementsPartial road closures are only allowed in light traffic areas where there will be at least 1 lane for traffic with extra traffic control measures to make sure 2way traffic can continue to run for the duration of the workAny change of traffic flow will require the proposal to be treated as a full road closureFor further information about a partial road closureplease refer to Contacts
If you are organizing a parademarch or other type of streetbased eventapproval is also required and we recommend you choose one of our 4 preferred routesChoosing an existing route reduces the amount of planning you will need to do
附:
悉尼道路封闭程序
量减少市中心悉尼外道路交通拥堵市高速公路部门会意日周六安排封路市中心外次道路会考虑日周六您必须申请进行道路施工道路相关活动砍伐树木超型工程机械梯子等批准请注意果您算阻塞道路行道需获批准
道路封闭申请必须预定日期前少月完成提交部门交通静委员会
道路部分全部关闭具体取决您求部分道路封闭允许少1条车道交通流量较少区域进行采取额外交通制措施确保2交通继续运行工程持续时间交通流量变化需提案视完全封路关部分封路更信息请参阅联系方式
果您组织游行游行类型街头活动需获批准建议您选择4条首选路线选择现路线减少您需做规划量
操作提示:通题干拉框选择题目正确答案
1As a ruleSydney's highway department will agree to road closures in the city centre scheduled for_________
[答案]CSundays
2If you want a road closed to trafficyou must complete your applications and send them to the relevant committee at least_________before the scheduled date
[答案]B30 days
3Partial of full road closures are decided by_________
[答案]Athe requirements from applicants
4Which is NOT the condition for partial road closures according to the passage?
[答案]BOn work days
5Which is the best title of this passage?
[答案]ARoad Closure Procedures in Sydney
附:
1作项规悉尼高速公路部门意计划_________关闭市中心道路
[答案]C星期日
2果您想条禁止通行道路您必须预定日期前少_________完成您申请提交相关委员会
[答案]B30天
3部分全封闭路_________决定
[答案]A申请求
4根通道属部分封路条件什?
[答案]B工作日
5篇文章佳标题什?
[答案]A悉尼封路程序
二阅读理解
阅读面文章根文章容判断文章句子正确(T)错误(F)(题10分)
BEIJING'S ROADS FOR THE FUTURE
  The road system in Beijing's central business district(CBD)is being redesigned to make public transportation easy and improve efficiency
  Plans for 45 roads within the CBD have been approvedmeasuring 415 kilometers in totalIn addition to public busesfree or low price shuttle buses from residence areas to subway stations will also be provided for commuters
  This is just over four times the number of roads opened in 2007and will add another 155 km to the existing road network
  Besides roadsan underground traffic system has also been operatingwhich includes four new subway lines beneath the CBD
  The underground connection system is made up of two levelsThe first underground floor is set aside mainly for pedestrians and commercial facilitieswhile the second underground floor is planned for parking
The city has also invested 140 million yuan to set up a traffic inquiry systemthrough which the public can check traffic conditions online and see the most convenient routes on their GPS systemIn the futurethe CBD will be fully covered by an intelligent traffic control system
附:
北京未路
北京中央商务区(CBD)道路系统正重新设计简化公交通提高效率
北京中央商务区45条道路规划已获批准总长415公里公巴士外通勤者提供住宅区铁站免费低价穿梭巴士
2007年开通道路数量四倍现道路网络中增加155公里
道路外交通系统已投入运营中包括北京中央商务区方四条新铁线
连接系统两层组成层行道商业设施预留二层规划停车场
该市投资14亿元建设交通查询系统市民通该系统线查询交通状况GPS系统查便捷路线未智交通全面覆盖北京中央商务区控制系统
操作提示:句子正确选择拉选项框T句子错误选择拉选项框F
1The road system in Beijing's central business district(CBD)is being designed again[答案]T
2Commuters can only take public buses from residence areas to subway stations[答案]F
3The redesigned road will be 155 kilometers long[答案]F
4The underground traffic system has four new subway lines beneath the CBD[答案]T
5Both levels of the underground connection system are planned for parking[答案]F
附:
1北京中央商务区(CBD)道路系统正重新设计[答案]T
2通勤者住宅区坐公巴士铁站[答案]F
3重新设计道路长155公里[答案]F
4交通系统北京中央商务区方4条新铁线[答案]T
5连接系统两层计划停车[答案]F
二阅读理解
阅读面文章根文章容判断文章句子正确(T)错误(F)(题10分)
ROAD CLOSURE PROCEDURES IN SYDNEY
  To minimize traffic jams in the city center and on major roads out of Sydneythe City's highway department will not agree to road closures scheduled for weekdays or SaturdaysOn minor roads outside the city centerweekdays and Saturdays may be consideredYou must apply for approval to do roadwork and other roadrelated activitiestree cutting and use of very large construction machineryladdersetcPlease note that approval is needed if you plan to block the road or footpath
  Applications to have a road closed to traffic must be completed and sent to the Department's Traffic Calming Committee at least a month before the scheduled date
  Roads can be closed partially or fullydepending on your requirementsPartial road closures are only allowed in light traffic areas where there will be at least 1 lane for traffic with extra traffic control measures to make sure 2way traffic can continue to run for the duration of the workAny change of traffic flow will require the proposal to be treated as a full road closureFor further information about a partial road closureplease refer to Contacts
If you are organizing a parademarch or other type of streetbased eventapproval is also required and we recommend you choose one of our 4 preferred routesChoosing an existing route reduces the amount of planning you will need to do
附:
悉尼道路封闭程序
量减少市中心悉尼外道路交通拥堵市高速公路部门会意日周六安排封路市中心外次道路会考虑日周六您必须申请进行道路施工道路相关活动砍伐树木超型工程机械梯子等批准请注意果您算阻塞道路行道需获批准
道路封闭申请必须预定日期前少月完成提交部门交通静委员会
道路部分全部关闭具体取决您求部分道路封闭允许少1条车道交通流量较少区域进行采取额外交通制措施确保2交通继续运行工程持续时间交通流量变化需提案视完全封路关部分封路更信息请参阅联系方式
果您组织游行游行类型街头活动需获批准建议您选择4条首选路线选择现路线减少您需做规划量
操作提示:句子正确选择拉选项框T句子错误选择拉选项框F
1Closures scheduled for Sundays on minor roads outside the city center may be considered[答案]F
2You need to get approval if you plan to block the road or footpath[答案]T
3Roads can only be closed partially[答案]F
4If you choose an existing route for a paradethe amount of planning will be reduced[答案]T
5Road closure applications can be sent to the Department's Traffic Calming Committee one week before the scheduled date[答案]F
附:
1考虑市中心外路周日关闭[回答]F
2果您算阻塞道路行道您需获批准[回答]T
3道路部分封闭[答案]F
4果您选择现路线进行游行计划数量减少[回答]T
5封路申请预定日期前星期送交署交通调静委员会[答案]F
单元测5 形考务试题答案
选择填空
[题目]_________is known to alltoo much fat causes heart problems
[答案]As
[题目]―Do you mind if I smoke here?
―__________________
[答案]Yesbetter not
附:
[题目]—介意里抽烟?
—__________________
[回答]
[题目]―Does she speak French or German?
―She doesn't_________
[答案]either
附:
[题目]—果邀请穆斯林吃饭什建议点菜?
—_____________
[答案]猪肉
[题目]―If you invite a Muslim to dinnerwhat are you advised not to order for him?
―_____________
[答案]pork
附:
1
[题目]―Jack won't like the filmyou know
―_________I don't care what Jack thinks
[答案]So what?
附:
[题目]—杰克喜欢部电影知道
—_________杰克想
[答案]样?
[题目]―What vegetables are in season now?
―Radish and carrot_________
[答案]I think
附:
[题目]―现什蔬菜时令?
—萝卜胡萝卜_________
[答案]认
[题目]―What do you think about Japanese food?
―____________________
[答案]Overallthe diet there is a healthy onelow fat
附:
[题目]—觉日菜样?
—____________________
[答案]总说饮食健康—低脂肪
[题目]A vegetarian diet consists of the following EXCEPT_________
[答案]meat
[题目]All things_________the planned flight will have to be can celled
[答案]considered
[题目]I_________a book when the telephone_________
[答案]was reading…rang
[题目]I don't think anyone can accuse him_________not being honest
[答案]of
[题目]I'm used to_________up early now
[答案]getting
[题目]The machine_________this morning for no reason
[答案]broke out
[题目]This is the book_________I have learned a lot
[答案]from which
[题目]We have spent all of our spare time_________our spare moneyon the project
[答案]as well as
二翻译
面句子选择正确翻译(题10分)
操作提示:通题干拉框选择题目正确答案
1Not only does the offending company suffer lossesbut the whole milk market suffers a sharp drop in sales
[答案]C仅违规公司遭受损失整奶制品市场销售额幅降
2But at the moment the industrial standards are too low to stop these problems from occurring
[答案]B目前行业标准太低法阻止问题发生
3A question that often troubles food companies is how to control problems like rats and insects without harming the food
[答案]B常困扰食品公司问题控制老鼠昆虫时会食物造成破坏
4The area of grass should be kept short to deny pests a place to live
[答案]A长草区域应该害虫生存空间
5Community service is a way for a person to give back to a community in which they live
[答案]B社区服务回报居住社区种方式
二听力理解
听话录音根话容判断正误(题10分)
TASK3 TESTMP3
操作提示:句子正确选择拉选项框T句子错误选择拉选项框F
1Helen is worried about food safety[答案]T
2So farthe contaminated baby milk hasn't caused any deaths yet[答案]F
3Food incidents occur not as often nowadays as before[答案]F
4Food safety is an old problem[答案]T
5Public confidence in the milk industry will be back soon[答案]F
附:
1海伦担心食品安全[回答]T
2目前止受污染婴奶粉没造成死亡[回答]F
3食品事件现前样常发生[答案]F
4食品安全老问题[回答]T
5公众牛奶行业信心快会恢复[答案]F
二听力理解
听话录音根话容判断正误(题10分)
TASK4 TESTMP3
操作提示:句子正确选择拉选项框T句子错误选择拉选项框F
1Michael is still drinking Koukou Old Yogurt these days[答案]F
2Industrial gelatin has reportedly been added into Koukou Old Yogurt[答案]T
3There are heavy metals in the yogurt under discussion[答案]T
4Food safety issues seem to be somewhat outstanding nowadays[答案]T
5At present people eat very little processed food[答案]F
附:
1迈克尔天喝口口老酸奶[答案]F
2报道口口老酸奶中添加工业明胶[回答]T
3正讨酸奶中含重金属[回答]T
4食品安全问题现似突出[回答]T
5目前少吃加工食品[回答]F
二听力理解
听话录音根话容完成方题目(题10分)
TASK4 TESTMP3
操作提示:通题干拉框选择题目正确答案
1The problem with Koukou Old Yogurt is that it contains_________
[答案]Cindustrial gelatin
2We can infer from the dialog that lead is_________
[答案]Aa heavy metal
3According to Michaelfood safety problems can't be prevented because_________
[答案]Cthe industrial standards are too low
4Melinda's solution to the food safety problem is_________
[答案]Apunishing the companies concerned severely
5In the dialog send a clear message to means to_________
[答案]Bwarn
附:
1口口老酸奶问题含_________
[答案]C工业明胶
2话中推断出角_________
[答案]A重金属
3根迈克尔说法食品安全问题法预防_________
[答案]C行业标准太低
4梅琳达食品安全问题解决方案_________
[答案]A严惩涉事企业
5话框中_________发送明确信息
[答案]Bwarn
二完型填空(空10分)
操作提示:通题目中拉选项框选择恰词语补全填空
Fruit is good for peopleMany people eat some{1}every dayMrBlack and MrsBlack like fruit very much and every Monday MrsBlack goes to buy some fruit in the shop near her houseThe man in the shop knows her well and helps a lotShe can buy all{2}of fruit therelike applespearsoranges and bananasIn different time of the year the{3}of each kind of fruit is not the{4}sometimes highsomething lowMrsBlack likes to buy cheap fruitBut MrBlack likes bananas onlyShe buys bananas for him every weekShe only buys cheap{5}for herself
附:
水果处{1}天吃布莱克先生布莱克夫非常喜欢水果星期布莱克夫会家附商店买水果店里男 解帮忙里买水果{2}苹果梨橙子香蕉年中时间{3}种水果{4}时高时低布莱克夫喜欢买便宜水果布莱克先生喜欢香蕉周买香蕉买便宜{5}
1[答案]fruit
2[答案]kinds
3[答案]price
4[答案]same
5[答案]fruit
二阅读理解
阅读面文章根文章容完成相应选择题(题10分)
FOOD SAFETY
  A question that often troubles food companies is how to control problems like rats and insects without harming the foodPest control experts say poisons should never be used where they may enter foodThey say the first line of defense against pests is to clean the places where the food is handled or stored
  Floorsfood preparation surfaces and tools can be disinfected with chemicals like chlorineiodine or bromineFood containers should be stored about half a meter off the ground and about half a meter away from any wallsStorage areas should be kept dry and clean
  Containers for raw materials such as ricewheatetcshould be kept closed and cleanAny foodstuff split during delivery or handling should be cleared up both inside and outside the buildingPests enter a building from the outsideso pay attention to outer wallsSmall stones can be used to cover the ground and block the growth of plants and the grass should be kept short to deny pests a place to live
  Check that doors and windows are completely closedAlso any hole that might allow pests to enter a building should be filledRats can enter through a hole the size of a coinA mouse can pass through a hole half that size
Supervisors should make a list of pest control measures to be taken by employeesThis will aid the efforts and also help prepare for any inspectionsIn most countriesthere is a food and drug administration bureauor some agency with a similar namewhich enforces rules nationwideresponsible for rules about the use of pesticidesetcHoweverbusinesses where food is storedprepared or sold must also follow state and local health lawsMost of these governmental departments or bureaus have their websites with information about food safety policies and other issuesPeople can easily access the information
附:
食品安全
常困扰食品公司问题损害食品情况控制老鼠昆虫等问题害虫防治专家说毒药永远应该进入食物方说付害虫第道防线清洁方处理储存食物方
板食物准备表面工具氯碘溴等化学物质进行消毒食品容器应存放离面约半米方距离墙壁约半米存放区域应保持干燥清洁
存放米麦等原材料容器应保持密闭清洁运送处理程中裂开食品应建筑物外进行清理害虫外部进入建筑物注意外部墙壁石头覆盖面阻挡植物生长草应该保持矮害虫没栖息
检查门窗否完全关闭害虫进入建筑物洞应该填满老鼠通硬币洞进入老鼠通半洞
应列出员工应采取害虫防治措施清单助工作助检查做准备数国家食品药品监督理局类似名称机构全国范围执行规负责关农药等规储存制备销售食品企业必须遵守州方卫生法律政府部门局中数网站提供关食品安全信息政策问题轻松访问信息
操作提示:通题干拉框选择题目正确答案
1According to pest control expertswhy should poisons never be used in food companies?
[答案]ABecause they may enter food
2What is the first line of defense against pests?
[答案]BTo clean the places where food is handled or stored
3How can floorsfood preparation surfaces and tools be disinfected?
[答案]CWith chemicals
4Why should attention be paid to outer walls?
[答案]ATo prevent pests from entering a building from outside
5What dose the passage mainly talk about?
[答案]CHow to control rats and insects without harming the food
附:
1害虫防治专家认什食品企业毒药?
[答案]A进入食物
2防虫第道防线什?
[答案]B清洁处理储存食物方
3板食物准备表面工具进行消毒?
[答案]C化学品
4什注意外墙?
[答案]A防止害虫外面进入建筑物
5文章讲什?
[答案]C伤害食物情况控制老鼠昆虫
二阅读理解
阅读面文章根文章容完成相应选择题(题10分)
FOOD SAFETY
  A question that often troubles food companies is how to control problems like rats and insects without harming the foodPest control experts say poisons should never be used where they may enter foodThey say the first line of defense against pests is to clean the places where the food is handled or stored
  Floorsfood preparation surfaces and tools can be disinfected with chemicals like chlorineiodine or bromineFood containers should be stored about half a meter off the ground and about half a meter away from any wallsStorage areas should be kept dry and clean
  Containers for raw materials such as ricewheatetcshould be kept closed and cleanAny foodstuff split during delivery or handling should be cleared up both inside and outside the buildingPests enter a building from the outsideso pay attention to outer wallsSmall stones can be used to cover the ground and block the growth of plants and the grass should be kept short to deny pests a place to live
  Check that doors and windows are completely closedAlso any hole that might allow pests to enter a building should be filledRats can enter through a hole the size of a coinA mouse can pass through a hole half that size
Supervisors should make a list of pest control measures to be taken by employeesThis will aid the efforts and also help prepare for any inspectionsIn most countriesthere is a food and drug administration bureauor some agency with a similar namewhich enforces rules nationwideresponsible for rules about the use of pesticidesetcHoweverbusinesses where food is storedprepared or sold must also follow state and local health lawsMost of these governmental departments or bureaus have their websites with information about food safety policies and other issuesPeople can easily access the information
附:
食品安全
常困扰食品公司问题损害食品情况控制老鼠昆虫等问题害虫防治专家说毒药永远应该进入食物方说付害虫第道防线清洁方处理储存食物方
板食物准备表面工具氯碘溴等化学物质进行消毒食品容器应存放离面约半米方距离墙壁约半米存放区域应保持干燥清洁
存放米麦等原材料容器应保持密闭清洁运送处理程中裂开食品应建筑物外进行清理害虫外部进入建筑物注意外部墙壁石头覆盖面阻挡植物生长草应该保持矮害虫没栖息
检查门窗否完全关闭害虫进入建筑物洞应该填满老鼠通硬币洞进入老鼠通半洞
应列出员工应采取害虫防治措施清单助工作助检查做准备数国家食品药品监督理局类似名称机构全国范围执行规负责关农药等规储存制备销售食品企业必须遵守州方卫生法律政府部门局中数网站提供关食品安全信息政策问题轻松访问信息
操作提示:通题干拉框选择题目正确答案
1Whereaccording to Pest control expertsshould poisons never be used?
[答案]BWhere there are few rats
2From the textwe know that like chlorineiodine or bromine are all_________
[答案]Cfruits
3Outer walls are built_________
[答案]Ato prevent pests from entering a building from outside
4Why is it necessary for supervisors to make a list of pest control measures?
[答案]CTo help control pests and help prepare for any inspection
5The responsibilities of a food and drug administration bureau is_________
[答案]Ato set up rules about the use of pesticides and enforces them all over the country
附:
1根害虫防治专家说法方毒药?
[答案]B老鼠少方
2文中知道氯碘溴等_________
[答案]C水果
3外墙建_________
[答案]A防止害虫外面进入建筑物
4什制定虫害防治措施清单?
[答案]C帮助控制害虫帮助准备检查
5食品药品监督理局职责_________
[答案]A制定农药规全国范围执行
二阅读理解
阅读面文章根文章容判断文章句子正确(T)错误(F)(题10分)
THE INTERNATIONAL FOOD SAFETY AUTHORITIES NETWORK (INFOSAN)
  With the rapid globalization of food production and tradeinternational issues related to food safety are likely to increaseFood safety authorities all over the world have realized that food safety problems should be carried out at both national and international levelsThere is a pressing need for exchanging routine information on food safety issuesand sharing experiences and information quickly when a food safety related event takes place
  INFOSAN is a joint program between the World Health Organization(WHO)and the Food and Agriculture Organization of the United Nations(FAO)This global network includes 181 Member StatesEach has a designated INFOSAN Emergency Contact Point which manages communication between national food safety authorities
  The network aims to:
  Promote the rapid exchange of information during food safety related events
  Share information on important food safety related issues of global interest
  Promote partnership and cooperation between countries
  Help countries strengthen their capacity to manage food safety risks
  As part of its routine activitiesthe INFOSAN Secretariat publishes reports in six languages and provides members with summaries and guidance on relevant food safety issues
  INFOSAN helps exchange food safety information and experience among its membersIt is also a useful resource for other public health organizations which recognize INFOSAN as an organization through which findings of investigations and strategies for improving food safety may be shared
INFOSAN monitors possible international food safety related events in close cooperation with the WHO Alert and Response Operations programINFOSAN can work with other international organizations to provide assistance to the countries or regions with emerging food safety events of possible international concern
附:
国际食品安全局网络(INFOSAN)
着食品生产贸易快速全球化食品安全相关国际问题会增加世界国食品安全部门已意识食品安全问题应该国家国际层面进行迫切需交流关食品安全问题日常信息发生食品安全相关事件时快速分享验信息
INFOSAN世界卫生组织(WHO)联合国粮食农业组织(FAO)间项联合计划全球网络包括181成员国国家指定INFOSAN紧急联络点负责理国家食品安全间沟通局
该网络旨:
促进食品安全相关事件中信息快速交流
分享全球关注重食品安全相关问题信息
促进国家间伙伴关系合作
帮助国加强理食品安全风险力
作日常活动部分INFOSAN 秘书处六种语言发布报告成员提供关食品安全问题摘指导
INFOSAN 帮助成员间交流食品安全信息验公卫生组织资源组织承认INFOSAN分享调查结果改善食品安全战略组织
INFOSANWHO警报应行动计划密切合作监测发生国际食品安全相关事件INFOSAN 国际组织合作出现国际关注新食品安全事件国家区提供援助
操作提示:句子正确选择拉选项框T句子错误选择拉选项框F
1It is probable that there will be more and more safety related issues worldwide[答案]T
2Food safety authorities in each country believe that food safety problems should be solved within their country[答案]F
3The major duty of the INFOSAN Emergency Contact Point in each FAO member state is to manage communication between national food safety authorities[答案]T
4Public health organizations can get findings of investigations and strategies for food safety improvement from INFOSAN[答案]T
5INFOSAN monitors possible international events concerning food safety independently[答案]F
附:
1世界范围会出现越越安全相关问题[回答]T
2国食品安全部门认食品安全问题应该国解决[答案]F
3FAO成员国INFOSAN紧急联络点职责理国家食品安全局间沟通[回答]T
4公卫生组织INFOSAN获食品安全改进调查结果策略[答案]T
5INFOSAN 独立监测国际食品安全事件[答案]F
单元测6 形考务试题答案
选择填空
[题目]―_____________
―To sum upit is a continual processfrom a newborn baby to the end of one's life
[答案]What does Community Health Service mean exactly?
附:
[题目]—_____________
—总言断程新生婴生命结束
[回答]社区卫生服务具体什意思?
[题目]―_____________
―Once or twice a week
[答案]How often will you arrange for me to visit someone at home?
附:
[题目]—_____________
—周次两次
[回答]会安排久回家次?
[题目]―_____________
―Generally speakingwe are concerned with all aspects of promoting good health
[答案]What health care programs are there in the community?
附:
[题目]—?
—_____________
[回答]外提供关抑郁焦虑教育
[题目]―Anything else?
―_____________
[答案]Moreoverwe provide education relating to depression and anxiety
附:
1世界
[题目]―Good morningVolunteering Mattershow can I help you?
―_____________
[答案]Good morningI'd like to find out how I could become a volunteer
附:
[题目]—早志愿服务重什帮助?
—_____________
[回答]早想知道样成名志愿者
[题目]Business people exercise their leadership in the community service are a_________the commercial world
[答案]as well as
[题目]Communities are part of everyday life and have positive_________on its members
[答案]effects
[题目]Community service is a great way to help people in their time of needIt is great to see the happiness that results_________it
[答案]from
[题目]Community service is important to me because it provides me with eyeopening experiences while ______ many people
[答案]benefitting
[题目]Community Services Program aims for all members of the community to_________to basic financial and material resourcesinformationadvice and referral
[答案]have access
[题目]Involvement in community activities_________an integral and inescapable part of senior managers' jobs
[答案]has seen as
[题目]It takes small steps to create big change and this is_________servicelearning hopes to achieve
[答案]what
[题目]Knowing_________increases employee loyalty to the company and their leader
[答案]that their CEO was volunteering time for community service
[题目]One of the main benefits of community service is that it_________a person's character
[答案]builds
[题目]The online What's on for Women provides information_________upcoming and ongoing events
[答案]about
二翻译
面句子选择正确翻译(题10分)
操作提示:通题干拉框选择题目正确答案
1Community service is a way for a person to give back to a community in which they live
[答案]B社区服务回报居住社区种方式
2Without community service people would not know the meaning of charity and giving back
[答案]C没社区服务会知道做慈善回馈意义
3People of all age groupswith a maximum number of youth are involved in the process of community service
[答案]A年龄段投身社区服务事业中中年轻
4Let's go to Citibank instead of the other guy down the street because they are good corporate citizens
[答案]B花旗银行吧街银行优秀公民企业
5It was the late 1970s that American manufacturing factories realized that quality control was a significant issue
[答案]C1970年代晚期美国制造业工厂意识质量控制重问题
二听力理解
听话录音根话容完成方题目(题10分)
TASK4 TESTMP3
操作提示:通题拉框选择题目正确答案
1What are health care programs mainly about?
[答案]All aspects of promoting good health
2What is the first part of community service health?
[答案]Communication
3How can people help themselves avoid illness?
[答案]Through diethealthy livingcutting out smoking and so on
4What areas are of growing concern?
[答案]Alcoholismdepression and anxiety
5Who typically provides bedside care?
[答案]The family
附:
1保健项目什?
[答案]促进身体健康方方面面
2社区服务健康第部分什?
[答案]通讯
3帮助避免疾病?
[答案]通饮食健康生活戒烟等
4领域越越受关注?
[答案]酗酒抑郁焦虑
5谁通常提供床边护理?
[答案]家庭
二完型填空(空10分)
操作提示:通题目中拉选项框选择恰词语补全填空
  Have you heard about Grandparents Week? This important community event began{1}a one day celebration in 2012 to commemorate National Grandparents DayIn 2014the ACT Government expanded the program to a whole week to fully{2}the true worth of grandparents in all their various forms
  Grandparents Week is an annual event to honor and recognize the valuable contribution grandparents makeand the valuable role they playin their families and the wider ACT community
  This year's festival includes an exciting range of eventsprograms and activities for young and old{3}those in between Sing some old time tunes or shake a move on the dance floorJoin a fun game of bowls or table tennis Play your hand at cardsAttend a Senior Club Open Day Partake in delicious morning teas{4}your vintage there's something for everyone during Grandparents Week
Does your community group have a great event or activity{5}for seniors? Would you like to include it as part of the Grandparents Week 2016 program?
附:
您听说祖父母周?重社区活动2012年开始{1}期天庆祝活动纪念全国祖父母日2014 年首领政府该计划扩整周充分{2}发挥祖父母真正价值种形式
祖父母周年度活动旨表彰表彰祖父母做宝贵贡献家庭更广泛 ACT社区中发挥宝贵作
年节日包括系列激动心活动节目活动适合年轻老年{3}介两者间唱古老曲子舞池中摇摇加入趣保龄球乒乓球赛玩牌参加高级俱乐部开放日享美味早茶{4}祖父母周期间适合您年份
您社区团体否适合老年型活动活动{5}?您想作2016年祖父母周计划部分?
1[答案]as
2[答案]capture
3[答案]plus
4[答案]Whatever
5[答案]in mind
二阅读理解
阅读面文章根文章容完成相应选择题(题10分)
DOORKNOCKING
  Doorknocking is the most effective way of making facetoface community contactbut it has become a lost artWith the recent focus on customer privacydoorknocking seems like an intrusion into other people's livesBut those who try it for the first time are usually surprised by the pleasant reception they receiveHere are some suggestions that will help you promote your services facetoface
BEFORE YOU GO OUT
  ◆BEGIN WITH A DOOR HANGER
  Try leaving a door hanger two days before you go doorknockingIt should briefly describe the project and say that someone will be around in person
  ◆WEAR AN OFFICIAL NAME TAG
  Doorknockers should wear name tags with the logo of their organizationThe best name tags will also include a color photo and the name of the caller
  ◆HAVE PEOPLE KNOCK THEIR OWN BLOCKS
  The easiest way to do doorknocking is to try your local area firstBeing a neighbor creates an immediate connection with the person answering the door after thateverything else is easy
  ◆FIND A GOOD EXCUSE TO DOORKNOCK
  A short survey about local concerns or a current project is a good excuseFigure out responses for various situationsWhat if the person who answers the door cannot speak English? What if a child answers the door?
  WHEN YOU GO OUT
  ◆WHEN TO GO AND WHAT TO DO FIRST
  The best times to knock are usually SaturdaysOn other daysafter dinner is idealWhen someone answers the doorsmile and introduce yourself say you are a volunteer andif it helpsstate where you liveGive the name of your organization andbrieflythe reason for the visit
  ◆STATE WHAT ACTION THE OTHER PERSON SHOULD TAKE AND THE BENEFITS
  Tell the person what they need to do and how their actions will benefit themselves and othersIf a person hesitatesemphasize benefits you've already mentioned and thenif necessaryadd further benefitsIf a person agreesfollow up immediatelyIf possibleget a donationa signatureor a promise
  ◆RECORD CONTACT INFORMATION ON THE SPOT
Record namesaddressesemail addressesphone numbersand responses to questions on the spotYou won't remember them later
附:
敲门
门面面社区联系效方式已成种失传艺术着客户隐私关注门似生活侵犯尝试第次通常会收愉快接感惊讶里建议帮助您面面推广您服务
出门前
◆门衣架开始
试着敲门前两天留门衣架应该简描述项目说会亲附
◆佩戴官方名牌
敲门应佩戴带组织标志姓名标签姓名标签包括彩色片电者姓名
◆敲砖
简单门方法先尝试您区作邻居应门者建立直接联系切容易
◆敲门找口
关问题前项目简短调查口找出种情况反应果开门会说英语办?果孩子开门办?
外出时
◆什时候先做什
敲门时间通常星期六日子晚餐理想应门时微笑介绍说名志愿者果帮助请说明住里说出名字组织简说明访问原
◆说明应该采取行动处
告诉需做什行受益果犹豫强调已提处然果必增加更处果意立进果获捐赠签名承诺
◆现场记录联系方式
现场记录姓名址邮箱址电话号码问题回答等会记
操作提示:通题干拉框选择题目正确答案
1Why does the author say that doorknocking has become a lost art?
[答案]BPeople focus on privacy increasinglyso doorknocking seems like an intrusion into other people's lives
2Which step should be firstly taken when you try doorknocking?
[答案]CLeave a door hanger
3What does the underlined phrase on the spot mean?
[答案]Awithout delay
4Which time is not appropriate for door knocking?
[答案]BMonday morning
5Who will receive pleasant reception when they try doorknocking?
[答案]CA neighbor who does good preparation and has significant project to introduce
附:
1什作者说敲门已成种失传艺术?
[答案]B越越注重隐私敲门似生活入侵
2尝试敲门时应先采取步?
[答案]C留门衣架
3现场划线句话什意思?
[答案]A毫拖延
4什时间适合敲门?
[答案]B星期早
5谁敲门会愉快接?
[答案]C准备充分重项目介绍邻居
二阅读理解
阅读面文章根文章容判断文章句子正确(T)错误(F)(题10分)
COMMUNITY SERVICE
  Community service is a service or activity which is performed by someone or a group of people for the benefit of the public or public organizations
  While volunteers may provide community servicenot everyone taking part in a community service program is a volunteerbecause not everyone is doing it through their own choiceSometimes people are asked to do it by:
  their governmentas part of a citizenship requirement instead of military service
  the courts and other criminal justice punishments
  their schoolto meet the requirements of a classor to meet the requirements of graduationorin the case of parentsrequired to provide a certain number of hours of service in order for their child to be enrolled in a school or sports team
  There are also people providing community servicebut they receive some form of payment in return for their work
YOUTH COMMUNITY SERVICE
  Community service performed by young peoplealso known as youth serviceis intended to strengthen young peoples' senses of civic engagement and communityand to help them achieve their educationaldevelopmental and social goals
HIGH SCHOOL GRADUATION
  Many educational institutions in the United States require students to perform community service hours to graduate from high schoolIn some high schools in Washington Statefor examplestudents must complete 60 hours of community service to receive a diploma
CONDITIONS OF PARTICIPATION
  Contribution of service is a condition of enrollment in some programsMost commonlyparents may be required to serve for their child to be enrolled in a school or sports team
COLLEGES
Though technically not a requirementmany colleges make community service an unofficial requirement for acceptanceHoweversome colleges prefer work experience over community serviceand some require that their students also participate in community service in order to graduate
附:
社区服务
社区服务某群公公组织利益进行服务活动
然志愿者提供社区服务参社区服务项目志愿者通选择做时会求样做:
政府作公民身份求部分服兵役
法院刑事司法处罚
学校满足班级求满足毕业求者父母说需提供定数量服务时间孩子参加学校体育运动团队
提供社区服务会收某种形式报酬换取工作
青年社区服务
青少年社区服务称青年服务旨加强青少年公民参感社区意识帮助实现教育发展社会目标
高中毕业
美国许教育机构求学生完成社区服务时间高中毕业例华盛顿州高中学生必须完成 60 时社区服务获文
参条件
服务贡献注册某计划条件常见情况父母需孩子提供服务进入学校运动队
学院
技术求许学社区服务作非官方录取求学更喜欢工作验社区服务学求学生参加社区服务毕业
操作提示:句子正确选择拉选项框T句子错误选择拉选项框F
1The organizer is the beneficiary of community service[答案]F
2Some people who take part in community services are not volunteers[答案]T
3Most of the youth who volunteer in community service are motivated by personal purposes[答案]F
4In some high schoolsstudents must complete certain hours of community service to receive a diploma[答案]T
5Every university considers community service is the most important experience for students among other practiceslike internship[答案]F
附:
1组织者社区服务受益者[答案]F
2参加社区服务志愿者[回答]T
3数志愿参加社区服务青年出目[答案]F
4高中学生必须完成定时间社区服务获文[回答]T
5学认社区服务学生实等实践中重历[回答]F
二阅读理解
阅读面文章根文章容选词填空(题10分)
COMMUNITY SERVICE
  Community service is a service or activity which is performed by someone or a group of people for the benefit of the public or public organizations
  While volunteers may provide community servicenot everyone taking part in a community service program is a volunteerbecause not everyone is doing it through their own choiceSometimes people are asked to do it by:
  their governmentas part of a citizenship requirement instead of military service
  the courts and other criminal justice punishments
  their schoolto meet the requirements of a classor to meet the requirements of graduationorin the case of parentsrequired to provide a certain number of hours of service in order for their child to be enrolled in a school or sports team
  There are also people providing community servicebut they receive some form of payment in return for their work
YOUTH COMMUNITY SERVICE
  Community service performed by young peoplealso known as youth serviceis intended to strengthen young peoples' senses of civic engagement and communityand to help them achieve their educationaldevelopmental and social goals
HIGH SCHOOL GRADUATION
  Many educational institutions in the United States require students to perform community service hours to graduate from high schoolIn some high schools in Washington Statefor examplestudents must complete 60 hours of community service to receive a diploma
CONDITIONS OF PARTICIPATION
  Contribution of service is a condition of enrollment in some programsMost commonlyparents may be required to serve for their child to be enrolled in a school or sports team
COLLEGES
Though technically not a requirementmany colleges make community service an unofficial requirement for acceptanceHoweversome colleges prefer work experience over community serviceand some require that their students also participate in community service in order to graduate
附:
社区服务
社区服务某群公公组织利益进行服务活动
然志愿者提供社区服务参社区服务项目志愿者通选择做时会求样做:
政府作公民身份求部分服兵役
法院刑事司法处罚
学校满足班级求满足毕业求者父母说需提供定数量服务时间孩子参加学校体育运动团队
提供社区服务会收某种形式报酬换取工作
青年社区服务
青年进行社区服务称青年服务旨加强青年公民参感社区意识帮助实现教育发展社会目标
高中毕业
美国许教育机构求学生完成社区服务时间高中毕业例华盛顿州高中学生必须完成60时社区服务获文
参条件
服务贡献注册某计划条件常见情况父母需孩子提供服务进入学校运动队
学院
技术讲项求许学社区服务作非官方录取求然学更喜欢工作验社区服务求学生参加社区服务毕业
操作提示:通题目中拉选项框选择恰词语补全句子
1I {1} our community because a great impact can be made on others' lives through simple actions
2{2}programs or activities are seen as opportunities for experiential learning
3Sometimes the government asks its people to serve their communities to meet the{3}
4Community service can be defined as a service that is performed for{4}
5Community service is a commitmentA person must want to do it with good intentions and not because they{5}do so
附:
1{1}社区通简单行动生活产生巨影响
2{2}计划活动视体验式学机会
3时政府求民社区服务满足{3}
4社区服务定义{4}执行服务
5社区服务种承诺必须怀着良意愿做{5}样做
1[答案]volunteer in
2[答案]community service
3[答案]citizenship requirement
4[答案]the benefit of
5[答案]are asked to
单元测7 形考务试题答案
选择填空
[题目]―How is everything going?
―_________we've been quite busy this month and there's been quite a lot of progress since the last inspection
[答案]As you can see
附:
[题目]—切进展?
―_________月忙次检查进步
[答案]您见
[题目]―I'd like to take a look first at those structural support beams that were going to be put in place on the second floor
―_________here's your jacket and helmetand you'd better put on these boots as wellit's our site policy
[答案]Certainly
附:
[题目]—想先安装二楼结构支撑梁
―_________夹克头盔穿双靴子—网站政策
[答案]然
[题目]―_________about it now?
―Every time a customer has complained we've followed our store policy and offered them an exchange or a full refund
[答案]What's being done
附:
[题目]―_________现呢?
―次客户投诉时会遵守商店政策提供换货全额退款
[答案]正做什
[题目]―I suggest you offer affected customers a discount on their next store purchaseexplain the situation and apologize to them
―Rightof course_________
[答案]I'll get on it right away
附:
[题目]―建议受影响客户次购买时提供折扣解释情况道歉
―然_________
[回答]马
[题目]―What's the latest situation with all these customer complaints we've been receiving?
―_________the quality of the material has not always been up to standard
[答案]I'm afraid
附:
[题目]―收客户投诉新情况什?
―_________材料质量总符合标准
[答案]害怕
[题目]A Chinese company is going to_________over the wellknown Japanese company TOSHIBA Corporation
[答案]take
[题目]Apple almost lost its dominance in the smartphone market_________it became clear that it was difficult to make a call using the much expected iPhone 4
[答案]when
[题目]I'm going to have a_________with Mark about this issue tomorrow
[答案]word
[题目]Let's go_________the reports on the latest work
[答案]over
[题目]Not only_________many cars built in the 1970s ugly and poorly designedthey also became very unreliable after 4050 thousand miles
[答案]were
[题目]Our new school building is_________construction
[答案]under
[题目]Proper controls that should_________developed as part of routine safety procedures were not in place
[答案]have been
[题目]The products of this company are always_________to standardthey have very strict quality control process
[答案]up
[题目]Then the situation_________worse by human error because of the lack of proper training
[答案]was made
[题目]Unluckilysome Quality Management System failures_________headlines
[答案]do make
二翻译
面句子选择正确翻译(题10分)
操作提示:通题干拉框选择题目正确答案
1Quality control is a series of activities that ensure and improve the quality of products and service
[答案]C质量控制系列确保提高产品质量服务行
2The goal of statistical quality control was to gather data that would allow for the constant improvement of manufacturing processes
[答案]A统计质量控制目标收集数便断改进制造工艺
3Involvementparticipationand teamwork are seen as absolute' must haves' if a quality workplace is to be created
[答案]C果高质量工作环境创造出绝必须条件视员工介入员工参团队合作
4It was the late 1970s that American manufacturing factories realized that quality control was a significant issue
[答案]C1970年代晚期美国制造业工厂意识质量控制重问题
5Doorways offer no greater protection than any other area
[答案]B出入口保护作区域强
二听力理解
点击听话录音根话容判断正误(题10分)
TASK4 TESTMP3
操作提示:句子正确选择拉选项框T句子错误选择拉选项框F
1Mike is met by an engineer at the construction site[答案]F
2The construction site doesn't look very busy[答案]F
3Mike doesn't inspect the whole construction site[答案]T
4It is not the first time the construction site being inspected[答案]T
5Fraser knows clearly how the quality is controlled at the construction site[答案]T
附:
1迈克建筑工遇位工程师[答案]F
2工起忙[回答]F
3迈克没检查整建筑工[答案]T
4施工现场第次检查[回答]T
5弗雷泽清楚知道施工现场控制质量[答案]T
二听力理解
听话录音根话容找出COLUMN A中方应产品质量问题(题10分)
TASK3 TESTMP3
操作提示:通拉框选择题目正确答案
COLUMN A
COLUMN B
1Customers:{1}
{Trying to understand and solve the problem Talking to the supplier Making complaints to the store Offering customers an exchange or a full refund Reporting the quality problem to the manager}
2The store:{2}
{Trying to understand and solve the problem Talking to the supplier Making complaints to the store Offering customers an exchange or a full refund Reporting the quality problem to the manager}
3The purchasing department:{3}
{Trying to understand and solve the problem Talking to the supplier Making complaints to the store Offering customers an exchange or a full refund Reporting the quality problem to the manager}
4The manager:{4}
{Trying to understand and solve the problem Talking to the supplier Making complaints to the store Offering customers an exchange or a full refund Reporting the quality problem to the manager}
5Frank:{5}
{Trying to understand and solve the problem Talking to the supplier Making complaints to the store Offering customers an exchange or a full refund Reporting the quality problem to the manager}
附:
A栏
B栏
1客户:{1}
{试图解解决问题供应商交谈商店投诉客户提供换货全额退款理报告质量问题}
2店铺:{2}
{试图解解决问题供应商交谈商店投诉客户提供换货全额退款理报告质量问题}
3采购部:{3}
{试图解解决问题供应商交谈商店投诉客户提供换货全额退款理报告质量问题}
4理:{4}
{试图解解决问题供应商交谈商店投诉客户提供换货全额退款理报告质量问题}
5弗兰克:{5}
{试图解解决问题供应商交谈商店投诉客户提供换货全额退款理报告质量问题}
1[答案]Making complaints to the store
2[答案]Offering customers an exchange or a full refund
3[答案]Talking to the supplier
4[答案]Trying to understand and solve the problem
5[答案]Reporting the quality problem to the manager
二听力理解
听话录音根话容完成方题目(题10分)
TASK3 TESTMP3
操作提示:通题干拉框选择题目正确答案
1What is the product they are talking about?
[答案]BShirts
2What is the problem with the product?
[答案]AQuality
3Who first finds out the problem?
[答案]BThe customer
4Who first receives the customer's complaints?
[答案]CThe store
5What is the source of the problem?
[答案]CThe supplier
附:
1说产品什?
[答案]B衬衫
2产品什问题?
[答案]A质量
3谁先发现问题?
[答案]B客户
4谁先收客户投诉?
[答案]C商店
5问题根源什?
[答案]C供应商
二完型填空(空10分)
操作提示:通题目中拉选项框选择恰词语补全填空
Total Quality Management(TQM)
Total Quality Management(TQM)is a{1}and structured approach to organizational management that seeks to improve the quality of products and services{2}ongoing refinements in response to continuous feedbackTQM requirements may be defined separately for a particular organization{3}may be in adherence to established standardssuch as the International Organization for Standardization's ISO 9000 seriesTQM can be applied to any type of organizationit originated in the manufacturing sector and has since been adapted for use in almost every type of organization imaginable{4}schoolshighway maintenancehotel managementand churchesAs a current focus of ebusinessTQM is based on quality management{5} the customer's point of view
附:
全面质量理(TQM)
全面质量理(TQM)种{1}结构化组织理方法旨提高产品服务质量{2}根断反馈断改进特定组织单独定义 TQM 求{3 }遵守定标准例国际标准化组织ISO 9000系列TQM 应类型组织起源制造业适想象类型组织{4}学校公路养护酒店理教堂作前电子商务重点TQM基质量理{5}客户观点
1[答案]comprehensive
2[答案]
3[答案]or
4[答案]including
5[答案]from
二阅读理解
阅读面文章根文章容判断文章句子正确(T)错误(F)(题10分)
NOTABLE QUALITY MANAGEMENT FAILURES
A Quality Management System(QMS)should clearly understand customer requirementsand to provide good designqualified production in an effective and efficient way
When the Quality Management System fails to fulfill its function and customer requirements are not metcustomers might be upset management and employees both might be unhappybut its negative influence is not so widespreadUnluckilysome Quality Management System failures do make headlines
Here are three cases of famous quality management failures:
3 MILE ISLAND
The worst nuclear disaster in UShistory broke out at the 3 Mile Island reactor facility in eastern Pennsylvania in March 1979Proper controls that should have been developed as part of routine safety procedures were not in placeThen the situation was made worse by human error because of the lack of proper trainingThe only new nuclear power stations built since that day were the ones already under construction
FORD PINTO
The Ford Pinto is the most wellknown and the worst disaster in the whole USautomobile industry in the 1970sApparently the entire USauto industry decided to spend the 1970s resting on their laurels after building great cars that sold well during the sixtiesNot only were many cars built in the 1970s ugly and poorly designedthey also became very unreliable after 4050 thousand milesThe Pinto tops the list of QMS failures because of its dangerous design faults and the questionable ethical decisions made by top management
IPHONE 4
Apple almost lost its dominance in the smartphone market when it became clear that it was difficult to make a call using the much expected iPhone 4Calls were dropped suddenly and frequently causing wide spread dissatisfaction among loyal customers who had upgraded to the new model early onIndependent tests showed that touching the left side of the case at a certain spot interrupted the signal and dropped the call― not a great feature for a handheld device mainly used to make callsMeanwhile Apple continued to deny or minimize the problemAs the problem continued to make news headlinesApple eventually agreed to a voluntary recall to correct the problem
All of the above famous quality management failures could probably have been prevented if the principles of a fully functioning quality management system had been in place
附:
显着质量理失败
质量理体系(QMS)应清楚解客户求效高效方式提供良设计合格生产
质量理体系未发挥作客户求满足时客户会高兴理层员工高兴负面影响广泛幸质量理体系失败确实头条新闻
著名质量理失败三案例:
三英里岛
美国历史严重核灾难1979年3月宾夕法尼亚州东部3英里岛反应堆设施爆发应作常规安全程序部分制定适控制措施没位然错误情况变更糟缺乏适培训天起建造唯新核电站已建
福特托
福特Pinto1970年代整美国汽车行业著名严重灾难显然整美国汽车行业制造60年代畅销伟汽车决定1970年代安现状1970年代制造许汽车丑陋设计佳行驶405万英里变非常Pinto危险设计缺陷高层理员做出问题道德决定位居质量理体系失败榜首
iPhone 4
备受期iPhone4电话变困难时苹果失智手机市场导位电话突然中断频繁引起早期升级新机型忠实客户广泛满独立测试表明某位置触摸外壳左侧会中断信号挂断电话 拨电话手持设备言功
时Apple 继续否认化问题作问题断成新闻头条苹果终意愿召回纠正问题
果功齐全质量理体系原已位述著名质量理失败避免
操作提示:句子正确选择拉选项框T句子错误选择拉选项框F
1A company should provide well designed products[答案]T
2A company should place company's interest at the first place[答案]F
3A company should reduce the amount of waste[答案]T
4A company should satisfy all employees' requirements[答案]F
5A company should produce qualified products[答案]T
附:
1公司应该提供设计良产品[回答]T
2公司应公司利益放首位[答案]F
3公司应该减少浪费[回答]T
4公司应该满足员工求[答案]F
5企业应该生产出合格产品[回答]T
二阅读理解
阅读面文章根文章容选择题目答案(题10分)
THE HISTORY OF QUALITY CONTROL
Quality control is a series of activities that ensure and improve the quality of products and service
The term' quality control' came into common use in the 1950s because of WEdward DemingHis' Fourteen Points' have become the bible for quality control specialistsWhen the postwar world returned to regular manufacturingDeming said that inspecting product quality after they were made was unacceptableInsteadhe proposed a process known as 'statistical quality control'It would use closely monitored performance measures to control quality while a product was being madeThe goal of statistical quality control was to gather data that would allow for the constant improvement of manufacturing processesThis would in turn improve quality control
Another important principle of Deming's beliefs was that upper management should be responsible for quality failuresHe strongly believed that it was possible for the workers to work hard to produce the highest quality products if they were given the right tools and working environmentIn Deming's own words'the basic cause of sickness in American industry and the resulting unemployment is a failure of top management to manage' He believed that strong leadership led to a motivated work force
If strong leadership is the term for managers in a quality environmentthen empowerment is the key concept for workers in Deming's systemImproved education and training are key factors in reaching employees and making them believe that their increased participation in the work process is an essential part of improving qualityInvolvementparticipationand teamwork are seen as absolute 'must haves' if a quality workplace is to be created
The Japanese were the first to use Deming's Fourteen Pointsand made great successAs an exampleone Japanese factory doubled production in just one year and was expecting to gain an additional 25 percent improvement the following yearwith no increase in the amount of working hoursAll this happened as a result of simply improving qualityIt was in the late 1970s that American manufacturing factories realized that quality control was a significant issueBy that timeJapanese firms such as Honda and Sony were taking over large portions of the American consumer market while it took most American firms until the 1990s to put quality control practices in place
附:
质量控制历史
质量控制确保提高产品服务质量系列活动
质量控制词1950年代爱德华•戴明普遍十四点已成质量控制专家圣战世界恢复正常制造时戴明说检查产品制造质量接受相反提出种称统计质量控制程会产品制造程中密切监控性测量控制质量统计质量控制目标收集数允许断改进制造程反会改善质量控制
戴明信仰重原层理员应质量失败负责坚信果工合适工具工作环境努力工作生产出高质量产品戴明换句话说美国工业生病失业根原高层理员理善相信强力领导会带积极员工队伍
果说强力领导优质环境中理者术语授权戴明体系中工关键概念改进教育培训接触员工相信更参工作程必少关键素提高质量部分果创建高质量工作场参参团队合作视绝必备条件
日率先戴明十四点取巨成功例家日工厂短短年产量翻番期第二年增加25产量没增加工作时间切发生提高质量直1970年代期美国制造工厂意识质量控制重问题时田索尼等日公司已接部分美国消费市场数美国公司直1990年代实施质量控制措施
操作提示:通题干拉框选择题目正确答案
1According to WEdward Demingwhat is the best time to control product quality ?
[答案]CDuring the production process
2When there are quality failureswhere does WEdward Deming think the blame lies?
[答案]CIn the tools
3Which of the following will not help to create a quality workplace?
[答案]AWorking alone
4During which period were some Japanese firms winning American consumer markets over American firms?
[答案]C1970s
5According to the passagewhich of the following statements about quality control is NOT correct?
[答案]BQuality control can be done within one department
附:
1根爱德华•戴明控制产品质量佳时间什时候?
[答案]C生产程中
2出现质量障时爱德华•戴明认责里?
[答案]C工具中
3项利创建优质工作场?
[答案]A独工作
4时期日企业战胜美国企业赢美国消费市场?
[答案]C1970s
5根文章列关质量控制说法正确?
[答案]B质量控制部门完成
单元测8 形考务试题答案
选择填空
[题目]_________these potential problemstwoway radios are preferable as they are extremely reliable for short distances and can broadcast to several people at once
[答案]Given
[题目]―_______________
―We have sent out leaflets to all residentsexplaining what to do in an emergency
[答案]What have you talked to the public?
附:
[题目]―_______________
―已居民发出传单说明紧急情况应措施
[回答]公众谈什?
[题目]―__________________
―With pleasureEmergency planning covers several different aspects
[答案]Could you explain how your work on emergency planning is going on?
附:
[题目]―__________________
―高兴应急计划包括方面
[回答]解释应急计划方面工作进行?
[题目]―__________________
―Firefire service please There's a huge fire here
[答案]911how may I direct your call?
附:
[题目]―__________________
―消防请消防 里火
[回答]911该转接电话?
[题目]―Hurry up If the fire spreads to the stairsit could block our way out
―_________This is no time for talking
[答案]You're right
附:
[题目]—快点 果火势蔓延楼梯会挡住路
—_________现说话时候
[答案]
[题目]―Perhaps we should climb out of the window and jump down? It's only two floors
―_________We'd better wait for the rescue
[答案]Nowe can't do that
附:
[题目]—许应该窗户爬跳?两层楼
—_________等救援
[回答]样做
[题目]A campus emergency_________occur at any time of the day or nightweekendor holidaywith little or no warning
[答案]may
[题目]Chemical spill guidelines should be established by the lab supervisor and should take into_________the following
[答案]consideration
[题目]Government agencies are responsible for declaring the evacuation of a given area and willin many instancesalready have evacuation plans_________
[答案]in place
[题目]Many disasters allow no time for people to gather even the most basic necessitieswhich is_________planning ahead is essential
[答案]why
[题目]Patrols should be conducted by teams_________individuals
[答案]rather than
[题目]Preparedness actions are aimed_________planningorganizingtrainingequippingexercisingevaluatingand implementing corrective actions to ensure effective coordination during incident response
[答案]at
[题目]Some of the risks may be relevant to your communityFind out which ones by_________wwwGetPrepareduk
[答案]visiting
[题目]These preparedness frameworks either exist in various stages of development_________have been implemented
[答案]or
[题目]When you complete Part 1you will be able to learn_________
[答案]what to do for specific hazards
二翻译
面句子选择正确翻译(题10分)
操作提示:通题干拉框选择题目正确答案
1Building occupants should not reenter the building until cleared by emergency personnel
[答案]C楼房居民危险情况没应急员清完前应该次进入楼道
2In additionit is recommended that all staff and faculty members maintain a personal emergency kit in their work area
[答案]A外建议员工工作区放置套急救包
3An emergency situation may result in recovery problems ranging from relatively minor to severe
[答案]A紧急情况会导致相轻微严重种恢复问题
4Doorways offer no greater protection than any other area
[答案]B出入口保护作区域强
5We will have to place ourselves where we can make the greatest contribution
[答案]C应该放做出贡献位置
二完型填空(空10分)
操作提示:通题目中拉选项框选择恰词语补全填空
  Over the decade 20012010an average of more than 700 natural and technological emergencies occurred globally every year{1}approximately 270 million people and causing over 130 000 deaths annuallyTwentyfive per cent of these emergenciesand 44 per cent of these deathsoccurred in less developed countries with limited capacities to prepare for and respond effectively{2}emergenciesThese statistics do not include the high levels of mortality and morbidity associated with conflictrelated emergencies{3}the World Bankover 15 billion people live in countries affected by violent conflictThese populations suffer from the consequences of societal disruption and increases in mortality and morbidity due to infectious diseasesacute malnutritiontrauma and complications from chronic diseases
  Over the same time periodrisks to public health{4}due to globalizationand international travel and tradeSuch risks might be transmitted by people (egSARSinfluenzapolioEbola)goodsfoodanimals(egzoonotic disease)vectors (egdengueplagueyellow fever)or the environment(egradionuclear releaseschemical spills or other contamination)
  In all types of emergenciesthe poorest and most vulnerable people suffer disproportionatelyThese{5}impacts are complicated by the enormity of the resulting economic costsaveraging over US100 billion per yearThe appropriate and timely management of these risks requires effective national and international capacitiesintersect oral collaborationthe promotion of equitythe protection of human rightsand the advancement of gender equality
附:
2001年2010年十年间全球年均发生700起然技术突发事件{1}约27亿年造成13万死亡中2544死亡中发生准备效应{2}紧急情况力限欠发达国家统计数包括突相关紧急情况相关高死亡率发病率{3}世界银行超15亿生活受暴力突影响国家口传染病急性营养良创伤慢性病发症遭受社会混乱死亡率发病率增加果
时期全球化国际旅行贸易公健康造成风险{4}风险通(例 SARS流感脊髓灰质炎埃博拉)商品食品动物(例畜患病传播)媒介(例登革热鼠疫黄热病)环境(egradionuclear释放化学泄漏污染)
类型紧急情况中贫穷脆弱遭受损失成例影响产生巨济成变复杂化均年超1000亿美元风险进行适时理需效国家国际力交叉口语合作促进公保护权促进性等
1[答案]affecting
2[答案]to
3[答案]According to
4[答案]have increased
5[答案]negative
二阅读理解
阅读面文章根文章容完成相应选择题(题10分)
EBOLA SURGE2014
  The 2014 Ebola outbreak is the largest in history and the first Ebola outbreak in West AfricaAlthough the current epidemic does not cause a significant risk to other nationsmany countriesincluding China and the United States of Americahave actually been working closely with the Ebola hit statesFor examplethe US Centers for Disease Control and Prevention (CDC) is taking precautions at home besides its activities abroad
  CDC' s team of virus hunters is supported by specialized public health teams both in West Africa and at the CDC Atlanta headquartersTogetherthey offer continuous support to save lives and protect peopleCDC works closely with a number of US government agenciesnational and international partnersCDC' s experience of working with Ebola is important to the World Health Organization's growing West Africa Ebola response
  On Sept22014CDC DirectorTom Frieden called for more international partners to join this effortThe sooner the world comes together to help West Africathe safer we all will beWe know how to stop this outbreakThere is a window of opportunity to do sothe challenge is to scale up the massive response needed to stop this outbreak
  CDC' s response to Ebola is the largest international outbreak response in CDC' s history with over 100 disease specialists on the ground in West Africasupported by hundreds of public health emergency response experts statesideactivated at Level 1its highest levelbecause of the significance of this outbreak
  The CDC supports affected countries to establish Emergency Operations Centers at national and local levels and helps countries track the epidemic including using realtime data to improve real time response
  Efforts in West Africa to identify those infected and track people who have come into contact with them are improvingThe CDC is operating and supporting labs in the region to improve diagnosis and testing samples from people with suspected Ebola from around the world
Local health care systems are strengthened through communicationcoordination with partners and training on infection control for health care workers and safe patient treatment
附:
埃博拉病毒激增2014
2014年埃博拉疫情历史规模次埃博拉疫情西非首次爆发埃博拉疫情然目前疫情国家没造成重风险包括中国美国许国家实际直密切合作例美国疾病控制预防中心(CDC)国外开展活动外国采取预防措施
美国疾病控制预防中心病毒猎手团队西非美国疾病控制预防中心亚特兰总部专业公卫生团队支持拯救生命保护民提供持续支持美国疾病控制预防中心许美国政府机构国家国际合作伙伴CDC埃博拉病毒合作验世界卫生组织日益增长西非埃博拉病毒应工作非常重
2014年9月2日疾病预防控制中心汤姆弗里登呼吁更国际合作伙伴加入努力世界越早团结起帮助西非越安全知道阻止次疫情样做机会窗—挑战扩阻止次疫情爆发需规模反应
美国疾病控制预防中心埃博拉病毒反应美国疾病控制预防中心历史规模国际疫情应措施100名疾病专家西非实工作美国数百名公卫生应急响应专家支持启动高级1级次爆发意义
美国疾病控制预防中心支持受影响国家国家方层面建立紧急行动中心帮助国追踪疫情包括实时数改进实时响应
西非识感染者追踪接触者努力正改善疾病预防控制中心正该区运营支持实验室改进诊断检测世界疑似埃博拉病毒样
通合作伙伴沟通协调医护员感染控制患者安全治疗培训方卫生保健系统加强
操作提示:通题干拉框选择题目正确答案
1How could we describe the 2014 Ebola outbreak?
[答案]CNot the first but the largest one
2The 2014 Ebola outbreak response was
[答案]Ban international activity
3What does the word massive mean in Para3?
[答案]AHuge
4How many disease specialists from CDC have gone to West Africa for the Ebola response?
[答案]CMore than 100
5What action does CDC take to respond to the large Ebola outbreak?
[答案]AWork closely with governmentalnational and international partners
附:
1形容2014年埃博拉疫情?
[答案]C第
22014年埃博拉疫情应
[答案]B国际活动
3第3段中量什意思?
[答案]A巨
4疾控中心少疾病专家赴西非应埃博拉?
[答案]C超100
5美国疾病控制预防中心针埃博拉爆发采取行动?
[答案]A政府国家国际合作伙伴密切合作
二阅读理解
阅读面文章根文章容判断文章句子正确(T)错误(F)(题10分)
HOW TO PREPARE AN EMERGENCY RESPONSE PLAN FOR YOUR WORKPLACE
  Emergencies and disasters can occur any time without warningThe more you are prepared for themthe better you will be able to actreducing panic and confusion when an emergency occurs
  Emergency response plans(ERPs)are intended to provide clear and effective steps for managing an emergencyAn ERP should deal with a variety of issues including evacuation of buildingscommunication with local emergency services and the general publicindividual roles and responsibilities during an emergencyand supplies and materials that may be needed
  Follow these basic guidelines to develop an effective ERP for your workplace:
  Determine when an evacuation would be necessary
  Choose primary and secondary evacuation routes and emergency exitsMake sure they are clearly marked and well litPost signs
  Install emergency lighting in case a power cut occurs during an evacuation
  Ensure that evacuation routes and emergency exits are wide enough for people to be evacuatedclear at all timesand unlikely to expose evacuating people to additional hazards
  Coordinate your plan with the local emergency management office
  Choose evacuation wardens who will help others during an evacuation and account for employees
  Establish specific evacuation proceduresThis includes a sample ERP to give you a better idea of what your procedures should include
  Establish a system for accounting for evacuated employees
  Hold emergency drills at least once a year to ensure that employees know what to do in an emergency and to test the effectiveness of emergency exit routes and proceduresKeep records of such drills
  Consider the transportation needs of employees
  Post evacuation procedures where employees can read them
  Establish procedures for helping people with disabilities and people who do not speak your local language
Consider how you could quickly get important personal information about employees in an emergency(for examplecontact numbers for their homenextofkinor special medical conditions)
附:
您工作场制定应急计划
突发事件灾难时发生毫预兆越做准备越采取行动减少紧急情况发生时恐慌混乱
应急响应计划(ERP s)旨理紧急情况提供清晰效步骤应急响应计划应处理种问题包括建筑物疏散应急服务公众沟通紧急情况角色责需品材料
请遵循基准您工作场开发效应急响应计划:
确定时需撤离
选择次疏散路线紧急出口确保清晰标记光线充足张贴标志
安装应急明防疏散期间发生断电
确保疏散路线紧急出口宽度足疏散始终保持畅通阻太疏散员面额外危险
应急理办公室协调您计划
选择疏散期间帮助员工负责疏散理员
建立具体疏散程序包括样应急响应计划您更解您程序应该包括什
建立撤离员核算制度
年少举行次应急演练确保员工知道紧急情况该做什测试紧急出口路线程序效性保留类演练记录
考虑员工交通需求
员工阅读方张贴疏散程序
制定帮助残障士会说语言程序
考虑紧急情况快速获取关员工重信息(例家庭亲特殊医疗状况联系电话号码)
操作提示:句子正确选择拉选项框T句子错误选择拉选项框F
1The emergency response plan is needed for reducing panic and confusion when an emergency occurs[答案]T
2Supply of basic necessities is not included in an ERP[答案]F
3According to the passagethe blocked emergency exits can help avoid further hazards[答案]F
4The evacuation plan of your workplace should be in line with that of local emergency management office[答案]T
5To ensure that employees know what to do in an emergency and to test the effectiveness of emergency exit routesthe emergency drills should be held once half a year[答案]F
附:
1应急预案减少紧急情况发生时恐慌混乱[回答]T
2基必需品供应包括企业资源计划中[答案]F
3根通道堵塞紧急出口帮助避免进步危险[答案]F
4您工作场疏散计划应应急理办公室致[回答]T
5确保员工紧急情况知道该做测试紧急出口路线效性应急演练应半年举行次[答案]F
文档香网(httpswwwxiangdangnet)户传

《香当网》用户分享的内容,不代表《香当网》观点或立场,请自行判断内容的真实性和可靠性!
该内容是文档的文本内容,更好的格式请下载文档

下载文档,方便阅读与编辑

文档的实际排版效果,会与网站的显示效果略有不同!!

需要 25 香币 [ 分享文档获得香币 ]

该文档为用户出售和定价!

购买文档

相关文档

1378国开电大本科《管理英语3》形考任务(单元自测1至8)试题及答案(精华版)

单元自测1 形考任务试题及答案一、选择填空[题目]―___________________―No,sorry,I didn't know the rule about punching in.[答案]Didn't you punch in this morning,Wendy?附:[题目]―___________________―不,对不起,我不知道打卡的规则。[回答]温迪,你今天早上没打卡吗?[题目]―I suppose there'll be a lot of arguments.

h***s 2年前 上传10946   0

国开(中央电大)本科《商务英语3》形考任务(单元自测1至8)试题及答案

题目顺序是随机的,使用查找功能(Ctrl+F)进行搜索[题目]— Why did you choose our company?— __________________________[答案] Sorry, I didn't catch that. Would you say it again?[题目]— Can you show me the schedule?— __________________________.[答案] Sure, a tight schedule as usual[题目]— But what do you want me to do there?— ______________________________

h***s 4年前 上传5765   0

1379国开电大本科《人文英语3》网上形考任务(单元自测1至8)试题及答案

说明:题目为随机,用快捷键查找功能搜索试题及答案。单元自测1 试题及答案一、选择题[题目]―Do you have much experience with caring for babies?―___________________[答案]Yes,I do.I often take care of kids in my free time.[题目]Lily is a good student except_________she is a little bit careless.[答案]that

h***s 2年前 上传1676   1

国开(中央电大)本科《人文英语3》形考任务(单元自测1至8)试题及答案

[题目]—Do you have much experience with caring for babies?—________.[答案]Yes, I do. I often take care of kids in my free time. [题目]Lily is a good student except ________ she is a little bit careless.[答案]that [题目]—How do you feel about your family life? —________.

h***s 4年前 上传16786   1

国开(中央电大)本科《管理英语4》形考任务(单元自测1至8)试题及答案

[题目] — This project is too big for me to finish on time.—________________.[答案]I'll give you a hand[题目] AT&T found that employees with better planning and decision-making skills were ______ to be promoted into management jobs.[答案]more likely[题目]______ CEOs spend planning, th

h***s 4年前 上传6721   0

「完整版」1378国开电大本科《管理英语3》网上形考任务(单元自测1至8)试题及答案

一、选择填空[题目]―___________________―No,sorry,I didn’t know the rule about punching in.A.Are you late this morning,Wendy?B.Didn't you punch out this morning,Wendy?C.Didn't you punch in this morning,Wendy?[答案]C[题目]―I suppose there’ ll be a lot of arguments.―__________________A.I should imagine so.B.No problem.C.That’s a good idea.[答案]A

h***s 1年前 上传726   0

国开(中央电大)本科《商务英语4》形考任务(单元自测1至8)试题及答案

[题目]— How would you like to go?—__________________.[答案]I'd like to fly to Frankfurt for a stay of two days[题目]____________ behalf of Vancouver C&S Int'l Trade Corp., I am pleased to invite you and your colleagues to visi

h***s 4年前 上传4527   1

国开(中央电大)本科《人文英语4》形考任务(单元自测1至8)试题及答案

[题目]– Do you think I can borrow your bike for a few hours?– ________________________________[答案]I'm sorry, but I really need it this afternoon.[题目]– Excuse me, could you tell the time?– ________________________________[答案]It's three thirty by my watch.[题目]– Good afternoon. Can I help you?

h***s 4年前 上传4400   0

1389国开电大本科《理工英语4》网上形考任务(单元自测1至8)试题及答案

题目为随机,用快捷键查找功能搜索题目及答案[题目]_________,he knows a lot about computer.[答案]Child as he is[题目]My assistant will now_________the machine in action.[答案]demonstrate[题目]_________this multimedia product has some shortcomings,it has also some good points.[答案]While

h***s 2年前 上传1458   0

「最新」国开(中央电大)本科《人文英语4》形考任务(单元自测1至8)试题及答案

[最新]国开电大本科《人文英语4》形考任务(单元自测1至8)试题及答案题目为随机,用查找功能(Ctrl+F)搜索题目单元自测1 试题及答案l [题目]– Do you think I can borrow your bike for a few hours?l – ________________________________l [答案]I'm sorry, but I rea

h***s 2年前 上传11802   0

国开(中央电大)本科《理工英语3》网上形考(单元自测1至8)试题及答案

[题目]— Could you please connect me with the HR Department?— _______[答案]Hold on a minute, please. I'll put you through.[题目]I have never read a book that is ______ than Harry Potter.[答案]more intereting[题目]The _____ question is much more different than this one.[答案]sixth

h***s 4年前 上传5385   0

2021春期1378管理英语3网上形考任务(单元自测1至8)试题及答案

单元自测1 形考任务试题及答案题目为随机,用查找功能(Ctrl+F)搜索题目[题目]— ______________________________— No, sorry, I didn't know the rule about punching in.[答案]Didn't you punch in this morning, Wendy?[题目]— I suppose there'll be a lot of arguments.— __________________[答案]I should imagine so.

h***s 3年前 上传270688   0

1377国开电大本科《理工英语3》网上形考任务(单元自测1至8)试题及答案(精华版)

1377国开电大本科《理工英语3》网上形考任务(单元自测1至8)试题及答案(精华版)说明:资料为国开平台形考和期末纸质考试的必备资料。单元自测1 形考任务试题及答案题目为随机,用快捷键查找功能搜索题目及答案[题目]―Could you please connect me with the HR Department?―___________________.[答案]Hold o

h***s 2年前 上传10396   0

1380国开电大本科《商务英语3》形考任务(单元自测1至8)试题及答案(精华版)

说明:资料为国开平台形考和期末纸质考试的必备资料。单元自测1 试题及答案题目为随机,用快捷键查找功能搜索题目及答案[题目]―Why did you choose our company?―___________________.[答案]Sorry, I didn't catch that.Would you say it again?附:[题目]―你为什么选择我们公司?―___________________。[回答]对不起,我没听懂。你会再说一遍吗?[题目]―Can you show me the schedule?

h***s 2年前 上传578   0

1379国开电大本科《人文英语3》网上形考任务(单元自测1至8)试题及答案(精华版)

一、选择题[题目]―Do you have much experience with caring for babies?―___________________[答案]Yes,I do.I often take care of kids in my free time.附:[题目]―你有很多照顾婴儿的经验吗?—___________________[回答]是的,我愿意。 我经常在空闲时间照顾孩子。[题目]Lily is a good student except_________she is a little bit careless.[答案]that [题目]―How do you feel about your family life?―___________________

h***s 2年前 上传806   0

国开(中央电大)专科《商务英语1》形考任务(单元自测1至8)试题及答案

说明:课程代码:04009。适用专业及层次:电子商务、工商管理(市场营销方向)、会计学(财务会计方向)、金融(保险方向)、金融(货币银行方向)、金融(金融与财务方向)、物流管理和物业管理专科。平台考试网址:http://www.ouchn.cn。单元自测1 试题及答案题目顺序是随机的,使用查找功能(Ctrl+F)进行搜索[题目]— Can you please tell me about your responsibilities?—_________________________________.[答案] Yes. I was in charge of the northwest region of China.[题目]— Could you give me an example of good team working there?—_________________________________.

h***s 4年前 上传8574   0

1389国开电大本科《管理英语4》形考任务(单元自测1至8)试题及答案(精华版)

1389国开电大本科《管理英语4》形考任务(单元自测1至8)试题及答案(精华版)说明:题目为随机,用快捷键查找功能搜索试题及答案;资料为国开平台形考和期末纸质考试的必备资料。单元自测1 试题及答案一、选择填空[题目]―This project is too big for me to finish on time.―_________________[答案]I'll give

h***s 2年前 上传48041   0

国开电大本科《管理英语3》在线形考(单元自测1至8)试题及答案

一、选择填空―___________________―No,sorry,I didn’t know the rule about punching in.A.Are you late this morning,Wendy?B.Didn’t you punch out this morning,Wendy?C.Didn’t you punch in this morning,Wendy?[答案]C―I suppose there’ ll be a lot of arguments.―__________________A.I should imagine so.B.No problem.

h***s 8个月前 上传382   0

3937国开电大专科《人文英语2》网上形考任务(单元自测1至8)试题及答案

一、选择填空题[题目]_________dangerous it is to ride fast on a busy road![答案]How[题目]_________pity you missed the lecture again![答案]What a[题目]―What do you think about my home town?―___________________

h***s 2年前 上传1410   0

3935国开电大专科《理工英语2》网上形考任务(单元自测1至8)试题及答案

题目为随机,用快捷键查找功能搜索题目[题目]―_________________.―I’m very glad to hear that.[答案]I’m sure to come on time[题目]―Do you like your present job?―_________,I just do it for a living.[答案]Not really[题目]―How was your job interview last Monday?―_________.She seemed interested in my working experience,but she didn’t ask for references.

h***s 2年前 上传655   0

国开(中央电大)专科《商务英语2》形考任务(单元自测1至8)试题及答案

题目顺序是随机的,使用查找功能(Ctrl+F)进行搜索[题目]—As you know, I’m not quite familiar with how to shop online.—It’s a piece of cake! I’ll show you how to do it. [答案]First of all, you should find a good e-commerce site.[题目]Consumers can’t digest too much information on a web page, resulting ______ low rates of return.[答案] in[题目]—Great! Maybe from now on, I should try to buy goods online.—________________.[答案] Why not? Go ahead. Just give it a try. [题目]I know personal selling is also a very effective tool for building ______buyers’ confidence.[答案] up

h***s 4年前 上传4203   0

国开(中央电大)专科《管理英语1》网上形考(单元自测1至8)试题及答案

说明:课程编号:04017。 适用专业及层次:城市轨道交通运营管理、工商管理(工商企业管理方向)、工商管理(市场营销方向)、旅游(酒店管理方向)、旅游(旅游管理方向)和行政管理专科。 考试平台:http://www.ouchn.cn。单元自测1 试题及答案题目为随机,用查找功能(Ctrl+F)搜索题目[题目]—____________________________.— Please call me Mary. That's my first name.[答案]How shall I address you?[题目]— Excuse me,— Yes, it's 8633-2788.If you have any other questions, don't hesitate to ask.[答案]could you please tell me what's the telephone number on my desk?[题目]— Hello, I'm David Chen. Nice to meet you.

h***s 4年前 上传12964   0

1389国开电大本科《理工英语4》网上形考任务(单元自测1至8)试题及答案(精华版)

1389国开电大本科《理工英语4》网上形考任务(单元自测1至8)试题及答案(精华版)说明:资料为国开平台形考和期末纸质考试的必备资料。单元自测1 试题及答案题目为随机,用快捷键查找功能搜索题目及答案[题目]_________,he knows a lot about computer.As he is a childChild as is heChild as he is

h***s 2年前 上传896   0

1391国开电大本科《商务英语4》形考任务(单元自测1至8)试题及答案(精华版)

1391国开电大本科《商务英语4》形考任务(单元自测1至8)试题及答案(精华版)说明:资料为国开平台形考和期末纸质考试的必备资料。单元自测1 试题及答案题目为随机,用快捷键查找功能搜索题目及答案[题目]―How would you like to go?―___________________.[答案]I'd like to fly to Frankfurt for a st

h***s 2年前 上传971   0

1390国开(中央电大)本科《人文英语4》形考任务(单元自测1至8)试题及答案(精华版)

一、选择题[题目]―Do you think I can borrow your bike for a few hours?―___________________[答案]I'm sorry,but I really need it this afternoon.附:[题目]—你觉得我可以借你的自行车几个小时吗?—___________________[回答]对不起,今天下午我真的需要它。[题目]―Excuse me,could you tell the time?―___________________[答案]It's three thirty by my watch.

h***s 2年前 上传71391   0